SlideShare une entreprise Scribd logo
1  sur  269
MOCK MAT / RMAT / CET  by :  DR. T.K. JAIN AFTERSCHO ☺ OL  centre for social entrepreneurship  sivakamu veterinary hospital road bikaner 334001 rajasthan, india FOR – PGPSE PARTICIPANTS  mobile : 91+9414430763
Be  Quicker faster accurate
Download material  http://www.4shared.com/file/129613569/c4e24e77/_2__MOCKCAT.html http://www.4shared.com/file/129077728/276ee61/CD_MOCK_MAT_QUIZ_MAY_2009.html http://www.4shared.com/file/129075830/b4e3d8a4/cdmat.html http://www.4shared.com/file/129078229/2bd90c4a/cd_write_mock_mat.html http://www.scribd.com/doc/27133626/Rmat-Mat-GMAT-Cat-Questions http://www.scribd.com/doc/23610014/Reasoning-e-Book http://www.4shared.com/file/129078352/f2882932/_2__cdmat.html
QUESTIONS ON BASIC MATHEMATICS  TURN TO NEXT SLIDES :....
Tea worth Rs 120 & 130  per Kg are mixed with a third variety in the ratio 1:1:2. If the mixture is worth Rs 150 per Kg , the price of the third variety per Kg will be?
SOLUTION  Let us assume that we use 1kg of first two varieties and 2 kg of 3 rd  variety. Total is 4 kg.  So the total price is : 150*4 = 600 price paide for 1 st  two varieties : (120+130) = 250 price paid for 3 rd  variety : 600-250 = 350 Price per kg for 3 rd  variety : 175 per kg.
A bill for Rs.6000 is drawn on July 14 at 5 months . It is  discounted on 5th October at 10%.Find the bankers discount  true discount, bankers gain and the money that the holder of  the  bill receives. Due date of bill : 17 december  interest for period (17 dec – 5 oct) = 73 days interst : 6000*73/365 * 10/100 = 120  this is banker's discount, the bank will give 5880 (money the holder will get )  formula for present value : 6000 * 100/ (100+rt) = 6000 *100/102 =5882.35  true discount is 117.65 answer
In one hour a boat goes 11 km with the stream and 5 km against the stream. The speed of the boat in still water is? The speed in still water is simple average of the two speeds. So it is ½ ( 11+5) = 8 Km.  So the speed of boat in still water is 8 km per hour.
A boat can travel with a speed of 13 kmph in still water. if the speed of stream is 4 kmph,find the time taken by the boat to go 68 km downstream? In downstream the speed of boat and stream will add up. 13+4 = 17 km per hour.  Thus 68/17 = 4 hours.  The boat will take 4 ours to cover a distance of 68 km downstream. Answer
A boat takes 19 hrs for travelling downstream from point A to point B. And coming back to a point C midway between A and B. if the velocity of the sream is 4 kmph . and the speed of the boat in still water is 14 kmph. what is the distence between A and B? The boat initially travelled at (14+4) = 18 km per hour. Then it travelled at (14-4) = 10 km per hour. Let us assume distance from A to B to be 2X. So it has travelled :  2x/18 + x/10 = 19 38X = 3420  x= 3420/38 = 90 so distance = 180 ans.
A boat takes 90 min less to travel 36 miles downstream then to travel the same distence upstream. if the speed of the boat in still water is 10 mph . the speed of the stream is : 90 minute = 1.5 hours Let us assume the speed of stream = x  36/ (10-x) – 36/ (10+x) = 1.5 so X = 2.
A can contains a mixture of two liquids A and B in the ratio 7:5 when 9 liters of mixture are drawn off and the can is filled with B,the ratio of A and B becomes 7:9. How many liters of liquid A was contained by the can initially? Total quantity is same before and after = X. The only change is in the ratio. 9 liters are taken out in ratio : 7:5, thus 9*7/12 = 5.25 is A and 3.75 is B.  We can write it as :  5/12 X + (9-3.75) = 9/16 X 5/12X -9/16X = -5.25 -7/48X = -5.25 or X = 360 the original quantity of A = 360 * 7/12 = 210 answer
.One quantity of wheat at Rs 9.30 per Kg are mixed with another quality at a certain rate in the ratio 8:7. If the mixture so formed be worth Rs 10 per Kg ,what is the rate per Kg of the second quality of wheat? Let us assume that the two types of wheat is mixed 8Kg and 7Kg respectively – total is 15 kg. Total cost : 15*10 = 150. Price paid for 1 st  type : 9.3*8 = 74.4 price paid for 2 nd  : 150 – 74.4 = 75.6  divide it by 7 to get price of 2 nd  : 75.6 / 7 = 10.8 answer
Tea worth Rs 120 and 130 per Kg are mixed with a third variety in the ratio 1:1:2. If the mixture is worth Rs 150 per Kg ,the price of the third variety per Kg will be? We have taken 1 kg of 1st, 1kg of 2 nd  and 2kg of 3rd. Total = 4 kg. Thus total price = 150*4 = 600.  price paid on 1 st  and 2 nd  = 250.  price paid on 3 rd  : 600-250 = 350  price per kg = 350/2 = 175 per kg. Answer
A vessel is filled with liquid,3 parts of which are water and 5 parts syrup. How much of the mixture must be drawn off and replaced with water so that the mixture may be half water and half syrup?  The ratio of water and syrup is 3:5.  total is 8.  we want them to be equal : 4:4.  thus we want syrup to be 4. now syrup is 5. thus we have to take 1 out of 5. Thus the ratio in which mixture has to be drawn out is 1/5. thus 8 * 1/5 = 1.6 must be drawn out, which in 3:5 is : .6 and 1  thus after drawing, the water and syrup is 2.4 and 4.  add 1.6 water again, and it is 4:4. so 1/5 th  of the mixture should be drawn off. Answer.
The price of an article including the sales tax is Rs 616.The rate of sales tax is 10%,if the shopkeeper has made a profit of 12%,then the cost price of the article is? Let us assume cost = 100 add profit(+12)  = 112  add sales tax (11.2) = 123.2 thus 100/123.2 * 616 = cost  = 500 answer
By selling 33m of cloth ,one gains the selling price of 11m.Find the gain percent? Suppose sale price is 1 per meter.  Total sale price = 33 profit = 11 cost = 33-11 = 22 thus profit is : 11/22 * 100 = 50% of cost. Answer
Find 3 numbers in the ratio : 2:3:5, the sum of whose squares is  608?  2x^2 +3x^2 +5x^2 = 608 38x^2 = 608 X^2 = 608/38 = 16  x= 4 numbers are 8, 12, 20  ans.
Download links ...for time and work problems  http://www.scribd.com/doc/24305787/Time-amp-Work-amp-Time-amp-Distance-in-Quantitative-Aptitude
Download links.. http://www.scribd.com/doc/6583802/1-July-Mathematics-Basics http://www.scribd.com/doc/11625711/Basic-Mathematics-1 http://www.scribd.com/doc/19628963/Mathematics http://www.scribd.com/doc/6683933/Probability-9-Sept http://www.scribd.com/doc/11626612/Probability http://www.scribd.com/doc/6683926/Probability-8-SEPT http://www.scribd.com/doc/25885859/Probability-for-Business-Management-Simplified http://www.scribd.com/doc/23301009/PROBABILITY http://www.scribd.com/doc/24894086/Basics-of-Permutation-and-Combination http://www.scribd.com/doc/7378688/6-August-Quantitative-Aptitude
Download links .... http://www.scribd.com/doc/23407732/22-jULY-BUSINESS-MATHEMATICS http://www.scribd.com/doc/23407815/28-July-Business-Mathematics http://www.scribd.com/doc/23407867/29-JULY-QUANTITATIVE-APTITUDE http://www.scribd.com/doc/7378688/6-August-Quantitative-Aptitude http://www.scribd.com/doc/23393694/14-June-aptitude-test-II http://www.scribd.com/doc/6583737/18-JUNE-Comprehensive-Test
Download links... http://www.scribd.com/doc/14675434/29-July-Quantitative-Aptitude http://www.scribd.com/doc/23393688/14-June-aptitude-test
Difficult words ... Turn to next slides ...
Download links  http://www.scribd.com/doc/6583519/English-Afterschoool-21-May http://www.scribd.com/doc/6906830/word-power-afterschoool http://www.scribd.com/doc/6583315/English-Improvement-Afterschoool http://www.scribd.com/doc/6583303/Reasoning-Afterschoool http://www.scribd.com/doc/6583520/English-Afterschoool-21-May-2 http://www.scribd.com/doc/6583519/English-Afterschoool-21-May http://www.scribd.com/doc/14660441/English-Afterschoool-23-May http://www.scribd.com/doc/22088672/Basic-Mathematics http://www.scribd.com/doc/19828297/Mathematics
Find the meaning of the following words ?  1.Sage : highly learned person  2.Salubrious : healthful 3.Sap : diminish , Undermine 4.Satiate : Satisfy fully 5.Savor : enjoy  6.Sear : char or burn ,  7. Scad : great quantity 8.Spate: sudden flood 9.Sodden : Soaked , dull  10.Snivel :  whine, to complain 11. Smirk : concited smile. 2.Slacken : slowup, loosen 13.Sineway: tough, setstrong and firm 14Shyster : lawyer using Questionable methods 15.Shard : fragment , generally of pottery 16.Stanch:Check flow of blood. 17.Stint : be thrifly, set limits 18.Stolid : dull , impassive 19. subside : settleddown, reduce 20.Sylvan: pertaining to the woods 21.Sybarite : lover of luxury.  22.Swindles : cheat
Difficult words and their meanings ... 1.Rankle=irritate,fester  2.Rancid=having bad odor 3.Raspy=harsh 4.Revage=plunder,despoil 5.Ravenous=extremely hungry 6.Realm=area of work 7.Renege=deny 8.Repast=meal,feast 9.Regal=royal 10.Rig=fix,manipulate  11.Roster=list 12.Reek=emit(odor) 13.Reiterate=repeat 14.Remorse=guilt,self-reproach 15.Regicide=murder of a king or queen 16.Refectory=dining hall  17.Redoient=fragrant,odorous, 18.Retinue=following,attendants 19.Ruse=trick 20.Ruffian=bully,scoundrel
How many times does the hands of a clock make right angle?  Let us take from 6 am  to 6 pm. At 6.15 it is almost right angle (not exact), similarly at about 6.45 also there is a right angle. Thus we have 2 occasions between 6 and 7. We should have 12 hours so 24 occasions when it can be in right angle. But between 9 to 10 we have only 1 such occasion (9.30), again from 3 to 4, we have only one such case. Thus in 12 hours we have only 22 right angles. Answer
Why is it so?  We can understand that by the time the minute hand covers 55 minutes, there is one hour (60 minutes). Thus there is a gap of 5 minutes. We can see that at times like 9 and 3 there is a problem. 9 am is counted in 8 to 9 (8.25 and 8.59) to count right angle.  And so is the case with 3 pm). If it is asked what is the exact time of right angle, use 60/11 as a formula to find exact time.
How many times do we have a straight angle between two hands of a clock from 6 am to 6 pm in a day?  Ideally it should be 12 (but it will not be 12) .  start at 6, 7.05, 8.10, 9.15, 10.20, 11.25, 12.30, 1.35, 2.40, 3.45, 4.50, 5.55, 6.  thus we have 13 such occasions when we have straight angle.
A watch which gains uniformly,is 5 min,slow at 8 o'clock in the morning on sunday and it is 5 min.48 sec.fast at 8 p.m on the following sunday. when was it correct? The watch has actually covered 10 min. 48 sec.  In a span of 180 hours. We want to know about the exact time when it had crossed only 5 minutes. So solve it using following formula :  5/ 10.8  (180) = 82.8  thus exact time is : 82/24 = 3 day + 10 hours Wednesday : 6 pm + 48 minutes. Answer
Explanation  Actually 180 is hours (from 8am to 8pm on next Sunday).  Actually 82.8 is in hours, to convert it into days, we have devided 82/24, we get 3 rd  day + 10 hours.  Why did we use : 10.  8 :  note: we converted 10.48 in decimal : 48/60 = .8 so it was 10.8
Questions on analogy ... Next slide .....
Errata : Books : : flaws:? (A)Manuscripts (B)Metals (C)Speech (D)Charter Metals (these are errors in flaws just like errata (errors ) in books)
Tractor : Trailer : : Horse : ? (A)Stable (B)Cart (C)Saddle (D)Engine Cart
Eye :Myopia : : Teeth : ? (A)Pyorrhea (B)Cataract (C)Trachoma (D)Eczema Pyorrhea (name of disease)
Fossils : Creatures : : Mummies : ? (A)Egypt (B)Human beings (C)Animals (D)Martyrs  Human beings
Reluctant : Keen : : Remarkable : ? (A)Usual (B)Restrained (C)Striking (D)Evolution  Reluctant = not interested keen = interested (so they are opposite)  the opposite of remarkable is usual
Sculptor : Statue : : Poet : ? (A)Canvas (B)Pen (C)Verse (D)Chisel  Sculptor makes statue Poet makes Verse
Pesticide : Crop : :Antiseptic : ? (A)Wound (B)Clothing (C)Bandage (D)Bleeding  Wound  pesticide is used to destroy antibodies in crop, the same role is played by antiseptic on wound
Engineer : Map : : Bricklayer : ? (A)Design (B)Template (C)Mould (D)Cement  Template
Fire : Ashes : : Explosion : ? (A)Flame (B)Death (C)Sound (D)Debris  Debris
Quail : Partridges : : Yak : ? (A)Cows (B)Deer (C)Oxen (D)Antelope  Similar to them is the relation between Yak and Oxen
Bank : River : : Coast : ? (A)Flood (B)Waves (C)Sea (D)Beach  Sea (Coast is the border of Sea, just like a bank is the border of river)
What is the opposite of the following words  Jaded (to mock / discourage) : Motivate / stimulate  .Jaundiced (biased)  = Unbiased .Jaunty (unbalanced, excited) =  sedate (calm) Jeopardy (to put in danger)  = Safety  Jettison (to throw out of ship)  x Salvage (to save)
Antonyms :  LACHRYMOSE (which produces tears) =  CHEERING (removes tears)  LACKADAISICAL (lazy)= AMBITIOUS LACONIC (in a few words) = VERBOSE (in lot of words)  LAMPOON (to ridicule) =PRAISE LANGUOR (in depression) = VITALITY (in youth and zeal)  LAVISH (who spends a lot) = FRUGAL (spending less)  .LAUDATORY (remarkable) =  DEFAMATORY (infamous) LECHERY (without character)= PURITY LETHARGIC (lazy)= INVIGORATING (full of zeal ) LEVITY (lightly)= SOLEMNITY (serious) LIMPID (clear) X TURBID (dirty) LOATH  (to hate)= EAGER (keen)  .LOQUACIOUS (talkative) = TACITURN (laconic) LUGUBRIOUS (mournful, sad)  =CHEERFUL  .LURID (colourful) = DULL
ANTONYMS    MALIGAN(to criticise) – Eulogize / praise MALLEABLE (ductile) = Brittle .MANIACLE (handcuff, chained) = Sane MANIFEST  (visible)= Obscure  MANUMIT (to free) = Enslave
Cobbler : Leather : : Carpenter : ? (A)Furniture (B)Wood (C)Hammer (D)Chair Wood
Rupee : Indian : : Yen : ? Japan
Ocean : Deserts : : Waves : ? (A)Sea (B)Dust (C)Sand dunes (D)Ripples Sand Dunes
Illiteracy : Education : : Flood : ? (A)Rain (B)Bridge (C)Dam (D)River Dam
Appraiser : Building : : Critic : ? (A)Book (B)Masterpiece (C)Judge (D)Gold Book
Dog : Bark : : Goat : ? (A)Bleat (B)Howl (C)Grunt (D)Bray Bleat
 Oxygen : Burn : : Carbon dioxide : ? (A)Isolate (B)Foam (C)Extinguish (D)Explode Extinguish
Water : Convection : : Space : ? (A)Conduction (B)Transference (C)Vacuum (D)Radiation  Radiation
Venerate : Worship : : Extol : ? (A)Glorify (B)Homage (C)Compliment (D)Recommend Venerate = worship = to pray  similarly, extol = to praise =  Glorify
Data sufficiency question... Next. ... slide ...
How is M related to N ?  1. P who has only 2 kids:  M& N. P is the  mother in law of Q, who is sister in law of N  2. R, the sister in law of M, is the daughter in law of S, who has only 2 kids – M and N.  From the first statement it is clear that M is Man. (M is husband of Q)  from second statement it is clear than N is also Man (N is husband of R).  thus M is brother of N  But for getting the answer, first statement is sufficient, so take care in exam, write only 1 st  statement is sufficient to answer.
How is T related to the man in photograph?  a. The man in photograph is the only son of T's grandfather b. The man in the photo has no brother / sister and his father is T's  Grandfather.  The answer can be drawn from either statement  (alone). Thus we can use any of these statements to answer the question.
two whole numbers whose sum is 64 cant be in the ratio ?  Options :  3:5,  1:7  3:4  9:7  answer : as you can see that the sum of 3 and 4 is 7, which cant divide 64, so this is the answer.
If a carton containing dozen mirrors is dropped, which of the following cant be the ratio of broken mirrors to unbroken mirrors?  Options :  2:1  3:1  3:2  7:5 again 3+2 is 5, which cant divide 12 so this cant be the ratio of broken to unbroken mirrors.  So answer is 3:2
A  father's age was 5 times son's age Five years ago. It will be 3 times after 2 years. What is the ratio of their present age?  Father = X, Son = Y Now  (X-5) = 5 (Y-5)  (from first statement)  (X+2)=3(Y+2) solving these two equations, we get :  X-5Y=-20  and X-3Y=4  , so Y = 12 X=40 thus their ratio is : 10:3 answer
Vinay got thrice as many marks in maths as in English. The proportion of marks in Maths and History is 4:3. If total marks in Maths, english, and History are 250, what are his marks in English? M: E = 3:1,  M:H = 4:3,  taking M as common and using cross multiplication,  M : E : H : 12: 4:9 total = 25 M = 12/25 * 250 = 120  E = 4/25 * 250 = 40 H = 9/25*250 = 90 answer
Data sufficiency question : out of which of the two option, can you get the answer ?  Question : 1. What is the rank of Mayank in the class ?  Options :  a. Swati is 5 th  from the bottom, and she is 30 places behind Mayank  b. There are 40 students after Mayank.
Solution  From the first statement, or from second statement, we can get the rank from bottom, but not from top. We dont know how many students are there in the class. So we are NOT able to get answer from any of these two statements. Answer
Data sufficiency question : from which of the following can we draw the answer ?  Question : What is the volume of the cubical pot?  Options :  1. the depth of the pot is equal to its width and the diagonal of the cubical pot is more than double the width.  2. The size of maximum size of ball that can be put in this pot is almost 80% of volume of this pot.
Solution  Answer cannot be drawn. None of the statement gives some help in finding volume of the cubical pot.
Data sufficiency question ?  Question :  How many Sons does X has ?  Options :  1. In the garden A was shouting X as father 2. X says that he has 1 offspring
Solution  None of the statement is of any help
Data sufficiency question  Question : How many persons are sitting in a roundtable?. Options : 1. There are 3 persons to the right of X  2. Y has 2 persons to his left and 2 to his right.
Solution  Answer cannot be drawn from either statement.
Data sufficiency question ?  Q : which direction is Ankit facing ?  Options :  1.  The rays of Sun are falling to the Right of Ankit.  2. It is morning time now.
Answer  Combining both the options, we can draw the answer. Ankit is North facing as his shadow is toward left and rays of Sun are towards right. .
Two equal glasses are respectively 1/3 and ¼ full of milk. They are then filled up with water and the contents mixed in a tumbler. The ratio of milk and water in the tumbler is ?  Let us assume that both the glasses have (3*4) = 12 liters of capacity. The first has 4 liters of milk and 2 nd  glass has 3 liters of milk. Combined together they have 7 liters of milk and 17 liters of water.  Thus the ratio of milk and water is : 7:17 answer
The incomes of A and B are in the ratio of 3:2, and their expenditures are in the ratio 5:3. if each saves Rs. 1000, what is the income of A?  Incomes are in ratio of X and expenditures are in the ratio of Y.  3X-5Y =  1000 2X-3Y=1000  -Y=-1000, X = 2000 thus incomes of A and B : 6000 and 4000 and expenditures are : 5000 and 3000 answer.
A sum of Rs. 1300 is divided between A,B,C,D such that A's share divided by B's share = B's share / C's share = C's share / D's share = 2/3  In such questions, start from Last – and take cube of the last number to start with – assuming that value for the last variable. (cube , if there are 3 iteration, if there are 4 iterations, take quadruple)  Let us start from D, assume it to be 27, now C is 18 now accordingly  : B is 12. now A is 8 the numbers are : 8, 12,18,27 thus A should get : 8/ 65 *1300 = 160
Find the third proportion to 9 : 12?  9:12 :: 12:X  multiply 9 by X  and 12 to 12.  9X = 144 X = 16 answer
Ankit looks at a photograph and says : “It is the photo of the only daughter of the maternal Grandfather of the  only son of brother in law of only son of my maternal grand father.” whose photograph was it ?  For this question, solve from last. Convert each clause into a single word and move forward.  Only son of my maternal granfather = Mama (maternal uncle)  Brother in law of Mama = Papa (father).  So on.... you will find the photo is that of Ankit's Mother. Answer
In a class out of 24 students 14 passed in Enlgish, 14 passed in Hindi, 12 passed in Both the papers. How many failed in both the papers?  For such questions, prepare a venn diagram to solve it . Answer is 8.  See it in next slide :
Total = 24, out of box = 24-16 =8 = failed in both.  12 English  Hindi  2 2 2
A:B = 2:3, B:C=5:7, find A:B:C  Here B is common. So cross multiply to solve it. Thus 5 must be multiplied to 2:3 and 3 must be multiplied to 5:7 answer : A:B:C= 10:15:21 answer
 
Mayank has 20 shares of RIL and market price of one share is 1060. How much money will he get by selling them. Brokerage rate is 1%?  Sale price = 1060*20 = 21200 less 1% brokerage = 212 net amount = 20988 answer
Atal does a work in 6 hours and Sonia does the same work in 9 hours. They work together. How much time will they take?  1/6 + 1/9 = (9+6) / 54 now multiply it by ½ ( as there are two persons here) : 15/54 * ½ =  15/108  now reverse it  : 108/15 = 36/5
A,B,C together complete a work in 6 days. A and B together do the same work in 10 days. How much time will C alone take to do the work?  1/6 – 1/10 = (10-6) / (10*6) = 4/60 = 1/15 reverse it : 15. so C alone will take 15 days to complete the work.
Divide 27x^3 – 8 y^3 by 3x-2y  (3x-2y)^3 = 27x^3 – 8 y^3  so remove its one power  we get : (3x-2y )^2 we get : 9x^2-6xy+4y^2 answer
Mayank shows a photograph and says that the mother of this person is the wife of the person who is father of the person, to whom a girl calls mother, the the brother of that girl calls Mayank his father. Whose photograph is that  (relation with Mayank)?  Mayank's brother in Law
Ram gives to Shyam all the money that he has in his pocket. Shyam adds that much more from his pocket and gives to Ravi, who further adds that much amount and gives that to Kavi, who triples this money and gives to Raju. Raju gives 10 times this amount to Mohan. Mohan gives 90% amount of this to Niru and retains Rs. 120. what is the ratio of this amount to Ram's original amount ?  Answer : 10:1 answer
The Emperor Augustus, it appears, commissioned an idealized sculpture portrait, the features of which are so unrealistic that they have constituted  what one scholar calls an "artificial face." - correct it  The Emperor Augustus, it appears, commissioned an idealized sculpture portrait, the features of which are so unrealistic as to constitute what one scholar calls an "artificial face."
A recent national study of the public schools shows  that there are now one microcomputer for every  thirty-two pupils, four times as many than there  were four years ago. - correct it  A recent national study of the public schools shows that there  is  now one microcomputer for every thirty-two pupils, four times as many as than there were four years ago.
Never before had taxpayers confronted so many changes at once as they had in the Tax Reform Act of 1986. Never before had taxpayers confronted so many changes at once as they in the Tax Reform Act of 1986. (as per past perfect tense, we are using had with the first part – so there is no need with the 2 nd  part)
Correct it : As have many self-taught artists, Perle Hessing did not begin to paint until she was well into middle age. Like many self-taught artists, Perle Hessing did not begin to paint until she was well into middle age.
Statements: I. All pilots are experts.  II. All authors are pilots. Conclusions: I. All authors are experts. II. No expert in an author. Only 1 st  conclusion is valid conclusion.
Statements: I. Some doctors are institutes. II. Some crooks are institutes. Conclusions: I. All institutes are doctors. II. Some institutes are crooks. Only 2 nd  conclusion is valid conclusion
If the positions of the fifth and the twelfth letters of the word GLORIFICATIONS are interchanged and likewise the positions of the fourth and fourteenth letters, the third and tenth letters, the second and eleventh letters, the first and thirteenth letters, are interchanged, then which of the following will be the twelfth letter from the right end? Answer : T
  Ostrich is related to Antelope in the same way as Egret is related to (a) Cow (b) Buffalo (c) Camel (d) Zebra Buffalo (both depend on each other)
  Hong Kong is related to China in the same way as Vatican is related to..?... (a) Canada (b) Mexico (c) North America (d) Rome
Solution  Hongkong is part of China, similarly Vetican is a part of Rome.
Forfeit is related to Surrender in the same way as Remit is related to (a) Perceive (b) Confiscate (c) Exempt (d) Refrain
Solution ... Refrain
  Who is paternal uncle of Pavan? I. Pavan is brother of Poornima, who is daughter of Meena, who is sister of Kumar, who is brother Smrithi. II. Prithvi is brother of Indrajith, who is husband of Poornima, who is mother of Ganga, who is sister of Pavan. From 2 nd  statement – it is clear that uncle of Pavan is Prithvi, but 1 st  statement doesnt give us this answer.
  What is Milan’s rank in the class of 44 students? I. Ramesh, whose rank is 17th in the class, is ahead of Shyam by 6 ranks, Shyam being 7 ranks ahead of Milan. II. Suketu is 26 ranks ahead of Milan and Shyamala is 6 ranks behind Milan while Savita stands exactly in the middle of Shyamala and Suketu  in  ranks,  her  rank  being 17th  Answer can be drawn from any statement.
  Four of the following five have similar relationship and hence form a group. Which one does not belong to the group? (a) BROTHER : DORVEHT (b) ENGLISH : GGNNSIJ (c) ANOTHER : CONVEHT (d) BETWEEN : DTEZEEP (e)  HUSBAND:JSUDNAF D is not proper. In Brother – two step to B, OR instead of RO, two step to T, EH instead of HE, two steps to R, we get DORVEHT. Similarly we have to solve all the options.
Which of the following relates to FLOWER in the same way as RTERBN relates to SECTOR? (a) RWLGPF (b) EOFKUQ (c) EOFMXS (d) RWLEND R then W, then L, now from behing, E instead of F, N instead of O, D instead of E. We get : RWLEND
  Who among the following British Governor- Generals shifted India’s capital from Calcutta to Delhi in 1911? a) Lord Louis Mountbatten b) Lord Canning c) Lord Hardinge d) Warren Hastings ANSWER : LORD HARDINGE
 “ Golden Handshake” is the term associated with a) Share market b) Retirement benefits c) Voluntary retirement benefits d) Smuggling VOLUNTARY RETIREMENT BENEFITSI
  Which of the following is the first surface- to- surface missile in India? a) Prithvi b) Trishul c) Agni d) Naag PRITHVI
  Mist is caused by a) Dry ice b) Ice at low temperature c) Water vapours at low temperature d) Carbon- monoxide in solid form WATER VAPOURS AT LOW TEMPERATURE
Who among the following was the author of “Rajtarangini”, commonly regarded as the first genuine history of India written by an Indian? a) Banbhatta b) Ravikirti c) Pushpadanta d) Kalhana KALHANA
  Which of the following Articles of the Indian Constitution deal with the Directive Principles of State Policy? a) 26 to 41 b) 31 to 56 c) 36 to 51 d) 41 to 66 36 TO 51
  The Chinese pilgrim Fa – Hien visited India during the reign of a) Kanishka b) Chandragupta I c) Chandragupta II d) Harshavardhana Chandragupta II
The Indian Navy’s only sailing ship, which returned to Kochi after a 10- month voyage around the globe is, a) INS Vibhuti b) INS Tarangini c) INS Prabhat d) INS Viraat INS Tarangini
Correct it : It being an important (a)/ letter, the draft had to be (b)/ seen by the Governor (c)/ itself for approval.  It being an important (a)/ letter, the draft had to be (b)/ seen by the Governor (c)/ himself for approval.
Correct it : Irrespective of either (a)/ Vijay or Sanjay goes (b)/ the overdue payment (c)/ cannot be collected. (d)/ No error.  Irrespective of whether (a)/ Vijay or Sanjay goes (b)/ the overdue payment (c)/ cannot be collected. (d)/ No error.
In a certain code language, ‘3a, 2b, 7c’ means ‘Truth is Eternal’; ‘7c, 9a, 8b, 3a’ means ‘Enmity is not Eternal’ and 6a, 4d, 2b, 8b’ means ‘Truth does not perish’. Which of the following means ‘enmity’ in that language? Is  and  eternal  are common in first two lines, so are 7c and 3a.  From the first two lines, we find that  enmity + not  means = 8b+9a from this and third sentence, we find  Not  is common, which is 8b. Thus 9a is left out, thus ENMITY = 9A (answer)
In a certain code language, ‘po ki top ma’ means ‘Usha is playing cards’; ‘Kop ja ki ma’ means ‘Asha is playing tennis’; ki top sop ho’ means ‘they are playing football’; and ‘po sur kop’ means ‘cards and tennis’. Which word in that language means ‘Asha’? In the first 2 sentences  IS PLAYING  are common,  ki ma  are also common.  Asha + tennis = kop + ja .  Let us compare this with  po sur kop , where  kop  is for  tennis  ( cards and tennis ). Thus  Asha = ja  (answer)
A long rope has to be cut to make 23 small pieces. If it is double folded to start with how many times does it need to be cut? Options : 9,12,11,10 11  as after cutting 11, we are able to get its double 22 pieces – and one last piece – which is at fold (one piece).
There are 19 hockey players in a club. On a particular day 14 were wearing the hockey shirts prescribed, while 11 were wearing the prescribed hockey pants. None of them was without either hockey pants or hockey shirts. How many were in complete hockey uniform? 14 + 11 – 19 = 6 answer
In a class room three fourth of the boys are above 160 cm in height and they are 18 in number. Also out of the total strength, the boys form only two third and the rest are girls. The total number of girls in the class is 18 are 3/4 th  of boys, so total number of boys is : 18 * 4/3 = 24 (total number of boys)  of the total strength the ratio of boys and girls is 2:1, so number of girls is 12. answer
‘A’ is east of ‘B’ and west of ‘C’. ‘H’ is South-West of ‘C’, ‘B’ is South-East of ‘X’. which is farthest West? When you put them on a map :  B  A  C (from west to East)  south to them is H  and X is further North West of B.  with the information given, it seems that X is the farthest West.
A girl earns twice as much in December as in each of the other months. What part of her entire year’s earning does she earn in December? Let us assume that she earns 1 in each month, but in December she earns 2. Total income = 11*1 + 2 = 13  thus she earns : 2/13 of her total income in December month alone.
One watch is 1 minute slow at 1 pm on Tuesday and 2 minutes fast at 1 pm on Thursday. When did it show the correct time? 48 hours have passed and in this duration the watch has gained 3 minutes. We want to know when the watch gained 1 minute. So it should be : 1/3 * 48 = 16 hours.  Thus the watch gave correct time on : 1+16 hours = 5 AM on Wednesday answer
Which number when placed at the sign of interrogation shall complete the matrix?  6  6  8 5  7  5 4  3  ? 120  126  320
solution Here you can see that when you multiply 6*5*4 , you get 120 and so on, thus you have to put 8 in place of interrogation sign (?) so make this matrix perfect. Answer.
In the sequence given below the sum of the two digits which immediately precede the digit ‘4′ exceeds the sum of the two digits which immediately follow the digit 4 and sum of the two digits which immediately follow the digit 6 exceeds the sum of the two digits which immediately precede the digit 6. How many such 4’s and 6’s together are there? 54462635642843766483 1) . Let us start with 4 after 54, sum of 54 is 9, and sum of 62 is 8, thus  our condition is fulfilled.  2) . 4 (56 =11, 28=10)  3) . 4 (66 * 83)  so 3 is answer (only 3 conditions by 4, no condition is fulfilled by any 6)
Below are given six three-digit numbers. The digits comprise ofnumeric and letters. The letter indicates its serial order in the English alphabet. What will be the middle digit of the 4th number when the numbers are arranged in the descending order after interchanging numeric in each number without altering the place of letter in the number? 19F, 2H9, 98B, D76, 7A6, 61E We have to interchange numeric first :  91F,9H2,89B,D67,6A7,16E now convert letters into numerics :  916,982,892,567,617,165 Now we have to put them in descending order : 982,916,892,617,567,165 the middle one of 4 th  number is  A (617 , 1=A) answer
In a code language any letter which is immediately after or before a vowel in the English alphabet is substituted by that vowel and any vowel i.e. A, E, I, O and U is substituted by the letter immediately following that vowel in the English alphabet. How can the word FEVERISH be written in that code language? FEVERISH : instead of F, write E, instead of E write F, U for V, F for E, no change in R, J instead of I, no change in S, I for H.  So answer :  EFUFRJSI answer
How many pairs of letters are there in the word SPONTANEOUS which have number of letters between them in the word one less than the number of letters between them in English alphabet? 1). P and T have gap of 2, in alphabet they have gap of : 3.  2). U and S have no gap, in alphabet they have gap of 1.  so answer is 2.
In the following question one term in the number series is wrong. Find out the wrong term. 11, 5, 20, 12, 40, 26, 74, 54 Here we have two series :  11*2 – 2 = 20 20*2 – 2 = 38 38*2 – 2 = 74 second series is :  5*2 + 2 = 12 12*2+2 = 26 26*2+2 = 54 so instead of 40, we should have 38. answer.
More material on reasoning :  Click here :  http://www.scribd.com/doc/23610014/Reasoning-e-Book
Mayank has three children — Sangeeta, Vimal and Ashish. Ashish married Monika, the eldest daughter of Mr. and Mrs. Roy. The Roys married their youngest daughter to the eldest son of Mr. and Mrs. Sharma,, and they had.two children named Amit and Shashi. The Roys have two more children, Roshan.and Vandana, both elder to Veena. Sameer and Ajay are sons of Ashish and Monika. Rashmi is the daughter of Amit. What is the surname of Rashmi ? Sharma (answer)
Six lectures A, B, C, D, E and F are to be organised in a span of seven days -from Sunday to Sunday ,-only one,lecture on each day in accordance with the following rules  (i)  A should not be organised on Thursday.  (ii)  C should be organised immediately after F.  (iii)  There should be a gap of two days between E and D.  (iv)  One day there will be no lecture (Friday is not that day), just before that day D will be organised.   (v)  B should be organised on Tuesday and should not be followed by D.  On which day there is no lecture ? Tentative schedule : S=d M=no T=b  W=e T=fF=c  S=a On Monday – there is no lecture.
There are five persons P, Q, R,S and T. One is football player, one is chess player and one is hockey player. P and S are unmarried and they dont participate in any game. None of the ladies plays chess or football. There is a married couple in which T is the husband. Q is the brother of R and is not a a chess player nor a hockey player. Who is the football player ?
Solution  P & S dont play so remove them. We have QRT left. One of them is lady, she must be R (as T and Q are men). T and Q play two games  - chess and football (as these are not played by ladies). Thus T plays chess and Q plays football and R plays hockey. Answer
Of the five boys A, B, C, D and E two are good, one is poor and two are average in studies. Two of them study in post-graduate classes and three in under-graduate classes. One comes from a rich family, two from middle-class families and two from poor families. One of them is interested in music, two in acting and one in sports. Of those studying in under-graduate Classes, two are average.and one is poor in studies. Of the two boys interested in acting, one is a post graduate student. The one interested in music comes from a middle-class family. Both the boys interested in acting are not industrious. The two boys coming from middle-class families are average in studies and one of them is interested in acting. The boy interested in sports comes from a poor family, while the one interested in music is industrious. E is industrious, good in studies, comes from a poor family and is not interested in acting, music or sports. C is poor in studies inspite of being industrious. A comes from a rich family and is not industrious but good in studies. B is industrious and comes from a middle-class family.  Name the boy interested in sports.
Solution  A=rich,non industrius , good in study,acting B=UGmiddle class, industrious , avg. In study ,music C= UGindustrious, poor in study (sports)  D=  UGmiddle, not-industrious,average in study (acting) E=poor,industrius,good in study, not -(M,S)
solution... Thus based on the table, we can say that C is interested in sports.
A girl leaves from her, home. She first walks 80 metres in North-west direction and then 30 metres in South-West direction Next, she walks,30 metres in South-east direction. Finally, she turns back towards her house. In which direction is she moving now ? Draw the map and solve it – the direction will be approximately EAST. ANSWER.
A walks 10 metres in front and 10 metres to the right. Then every time turning to his left, he walks 5, 15 and 15 metres respectively. How far is he now from his starting point ? DRAW THE MAP AND SOLVE IT  ANSWER = 5 METERS.
A child is looking for his father. He went 90 metres in the East before turning to his right. He went 20 metres before turning to his right again to look for his father at his uncle's place 30 metres from this point. His father was not there. From here he went 100 metres to the North before meeting his father in a street. How far did the son meet his father from the starting point ? DEDUCT 30 FROM 90 AND 20 FROM 100 FIND SQUARES OF 60 AND 80 AND ADD THEM, YOU GET 10000, SQ.ROOT OF WHICH IS 100. ANSWER IS 100 M.
A father tells his son, "I was of your present age when you were born." If the father is 44 now, how old was the boy 5 years back ? THE AGE OF SON IS 22 YEARS NOW. SO HIS AGE 5 YEARS BACK WAS 17 YEARS. ANSWER
In a certain code '13' means 'stop smoking' and '59' means 'injurious habit' What is the meaning of '9' and '5' respectively in that code ? I. '157' means 'stop Bad habit'. II., '839'means 'smoking is injurious'. How do you get answer ?  Solution : solution can be obtained from any of these two hints. If you take first hint, 59 and 157 have 5 as common number. The common number denotes habit. Thus 5 means Habit.  If you take 2 nd  hint, 9 is common between 59 and 839 and the common word is injurious, thus 9 means injurious. Thus you can use either of these two statements to answer.
WHAT IS THE PUNCHLINE OF BANK OF INDIA  RELATIONSHIP BEYOND BANKING
WHAT IS THE PUNCHLINE OF CITIBANK  CITI NEVER SLEEPS
WHAT IS THE PUNCHLINE OF  KOTAK MAHINDRA BANK  LETS MAKE MONEY SIMPLE
WHAT IS THE PUNCHLINE OF  SBI  WITH YOU ALL THE WAY
WHAT IS THE PUNCHLINE OF  DENA BANK  TRUSTED FAMILY BANK
WHAT IS THE PUNCHLINE OF  INDIA INFOLINE  ITS ALL ABOUT MONEY HONEY
MATCH THE CEO AND THE COMPANY  ,[object Object],[object Object],[object Object],[object Object],[object Object],[object Object],[object Object],[object Object],[object Object],[object Object]
WHO IS CEO OF KOTAK MAHINDRA BANK ?  UDAI KOTAK
WHO IS CEO OF HSBC NAINA LAL KIDWAI
WHO IS CEO OF ICICI CHANDA KOCHAR
WHO IS CEO OF HDFC BANK  ADITYA PURI
WHO IS CEO OF  AXIS BANK (FORMERLY UTI BANK)  SHIKHA SHARMA
WHO IS CEO OF STANDARD CHARTERED BANK  PETER SANDS
WHO IS CEO OF MORGAN STANLEY MUTUAL FUND N. RAMCHANDRAN
WHO IS CEO OF INDUSIND BANK  RAMESH SOBTI
WHO IS CEO OF MUTHOOT GROUP M. GEORGE MUTHOOT
WHO IS CEO OF YES BANK RANA KAPOOR
WHO IS CEO OF SBI? O.P. BHATT
WHO IS CEO OF BARKLEYS BANK MARCUS AGIUS
WHO IS CEO OF ABN AMRO BANK  GERRIT ZALM
WHO IS CEO OF ALLAHABAD BANK  OMKAR NATH SINGH
WHO IS CEO OF BANK OF AMERICA  BRIAN MOYNIHAN
WHO IS CEO OF BANK OF RAJASTHAN  G. PADMANABHAN
WHO IS CEO OF CANARA BANK A.C. MAHAJAN
WHO IS CHAIRMAN OF BANK OF MAHARASTRA  ALLEN PAREIRA
WHO IS CHAIRMAN OF ANDHRA BANK  K. RAMCHANDRAN
 
 
A, B, C, D, E, and F are a group of friends. There are two housewives, one professor, one engineer, one accountant and one lawyer in the group. There are only two married couples in the group. The lawyer is married to D, who is a housewife. No woman in the group is either an engineer or an accountant. C, the accountant, is married to F, who is a professor. A is married to a housewife. E is not a housewife. What is E's profession?  Males :C (accountant), A, (E) Lawyer  females : D(housewife),F (professor), (B) Housewife Thus E is a lawyer, as all other professions are occupied.
Five women decided to go shopping to The Loot Bikaner. They arrived at the the Loot showroom in the following order: 1. Archana, 2. Chellamma, 3. Dhenuka, 4. Helen, and 5. Shahnaz. Each woman spent at least Rs.1000. Below are some additional facts about how much they spent during their shopping spree.   i. The woman who spent Rs. 2234 arrived before the lady who spent Rs. 1193.   ii. One woman spent Rs. 1340 and she was not Dhenuka.   iii. One woman spent Rs. 1378 more than Chellamma.   iv. One woman spent Rs. 2517 and she was not Archana.   v. Helen spent more than Dhenuka.   vi. Shahnaz spent the largest amount and Chellamma the smallest. Questions : The woman who spent Rs. 1193 & when did she come?  A=1340, C = 1193, S=2573, H= 2517,D=2234  1193 is smallest, so it was spent by Chellamma. She is 2 nd  to come.  Answer
Let A and B be two solid spheres such that the surface area of B is 300% higher than the surface area of A. The volume of A is found to be k% lower than the volume of B. The value of k must be
solution Surface area of sphere : 4pi*r*r if we assume smaller sphere of R=1 and larger sphere of R=2, then our condition is fulfilled. Now find volume of the two spheres :  4/3*pi*r*r*r  so the ratio of the bigger and smaller spheres are : 8:1 thus volume of A is lower by : 7/8*100 = 85% anwer
There are 8436 steel balls, each with a radius of 1 centimeter, stacked in a pile, with 1 ball on top, 3 balls in the second layer, 6 in the third layer, 10 in the fourth, and so on. The number of horizontal layers in the pile is Solution : first pile : 1, 2 nd pilw ; 6 3rd pile :  let us add the series : 1+3+6+10+15+21+28+36...... there are two series : 1, 1+2, 3+3,6+4  .... ,
The 288th term of the series a, b, b, c, c, c, d, d, d, d, e, e, e, e, e, f, f, f, f, f, f.... should be : We can see the series is : 1,3,4, (a is 1, b is 2, c is 3 ,,, so on apply formula of Total of arithematic progression:  Tn = n/2(2A+(n-1)d) 288=n/2 (2+nd-d) use options to solve questions earliers.
A shopkeeper increases price of an item by 20% and then gives a discount of 20% on new price. What is the net impact ?  Let us assume that initial price is 100, new price is 120 now 20% discout is applied from 120 120 -24 = 96 thus we can see that there is a net impact of -  4%.  in short : 20% multiplied to 20% gives -4% ans.
There are certain words written randomly, can you infer what will come next :  A8R  L24M X72H ....?  Try to find some pattern :  A = 1, L = 12,  X = 24 next : K  numberic : 8 * 3 = 24, now 24*3 = 72, next :216  R = 18, M = 13,  H 8, next : C  we can say next code : K216C answer
We use + for multiply, - for divide, / for subtraction and * for addition. Following BODMAS, what should be the answer :  for the following :  3* 5-1/4+2 =  answer : 0  2* 3* 4/ 2 – 1 +2+2 = =1 answer  2-2+3-4/ 4 *2 = =- 5.25 answer
20 persons came to a party and each person shook hand with each other person. How many handshakes were there?  Start with small number to validate and understand. Suppose there are 4 persons, first will shake hand with 3, second will shake hand with 2, third will shake hand with 1. we have : 3+2+1 = 6. thus we can say : N * (N -1) / 2= 6 similarly here : (20 * (20-1)/ 2 = 190 answer
How many zeroes are there when you count from 1 to 100?  One each from 10 to 90 : + 2 in 100 = 11 zeros.
You take up all those positive numbers which are ending with 2 and are less than 100 and are divisible by 3. How many such numbers are there?   12, 42, 72  thus we have 3 such numbers.
We have 0,1,2,3,4 as digits. How many 5 digit numbers can we make out of these. Repeatition is not allowed?  For the first number, you have 4 options (zero not be used)  :  for 2 nd  number you have 4 options again (the first number will not be used) , for 3 rd  number we have 3 options, so on = 4*4*3*2*1  = 96 answer
Raju had to add two digits, but by mistake he multiplied these two digits and got 56. He was again asked to add these two digits, but this time he squared up these numbers and found their difference, it was 15. what will you get if you find  the correct answer ?  X* Y = 56 X^2 – Y^2 = 15 (x+Y) (X-Y) = 15  now try with options at this stage. Take factors of 56, which are : 2*2*2*7  the factors could be : 4*14 or 8*7  so X = 8 and Y = 7, their difference is 1 and X +Y = 15, so the correct answer is 15. answer  .
In order to go to his office from home, Vivek goes first to his right then he turns  towards left three times. Then he again turns to his right. He has to again turn left 2 times to enter his office, which is North facing. What direction is his house facing?   Total left turns : 3 + 2 = 5 total right turns : 1 + 1 = 2 net : 3 left turns.  Now from office, he will take net 3 right turns to enter his home. He will be west facing while entering home, so home will be East facing. Answer : east facing.
Kapil says that his birthday is after 26 February, but his friend Mayank says that it is before 1 March. His another friend Vivek says that it is between 25 and 28. When is the birth day of Kapil?  27 February. Answer
Mayank swims across a river a distance of 200 meters 12 minutes, but he can also swim with the current in water same distance in 5 minutes. What is his speed in still water?  Formula  = (Distance ) / (Time ) = speed  while going accross the river, the speed is : X-Y= a ( 200/12)= 16.66 , where X is Mayank's speed and Y = water's speed while with the water, the speed is X + Y = b (200/5) = 40  the speed of Mayank is average of 40 and 16.66 = 28.33 and the speed of water is (40-16.66)/2 = 11.66 m. Per minute. answer
WHO HAS TAKEN HIGHEST NUMBER OF WICKETS IN 1 DAY AND TEST CRICKET?  M . MURALIDHARAN OF SRILANKA
WHERE WILL NEXT OLYMPICS BE HELD?  LONDON 2012
NAME OF CAPTAIN OF HOCKEY TEAM IN INDIA ?  MALE : SANDEEP SINGH COACH : MAHARAJ KISHAN KAUSHIK  FEMALE : SURAJ LATA
WHICH COUNTRY PRODUCES MAXIMUM GOLD ?  SOUTH AFRICA (AND INDIA CONSUMES THE MOST)
WHO IS THE PRIME MINISTER OF UK?  TONY BLAIR
HOW MANY COUNTRIES ARE MEMBERS OF WTO NOW ?  153 WTO = World Trade Organisation , earlier called GATT, set up in 1995.
What is the nickname of Glen McGrath?  Pigeon
A,B,C speak one statement true and one false. A says to B : “I am a thief.” 2. “You are a Doctor.” B: “Yes, I am a doctor”  “C is a thief.”  C adds : 1. “ I am a theif.” “B is a Doctor.” Who is theif ?  Let us suppose A's 1 st  statemet is right, 2 nd  statement is wrong. Thus B's 1 st  statement is false and 2 nd  right. C's 2 nd  statement is false, thus 1 st  is correct. Here there are two theifs – A and C.  If we assume A's 2 nd  statement to be correct, then A is not a theif, and no one is a theif.  Thus either no one is theif or A+C both are theives. Logically , NO ONE is theif here.
A had to buy some  nuts. He buys half the nuts required. He takes them and his son steals half of these. Out of remaining, his daughter steals half. Now when A  is fired by his boss that he had brought exactly 70 less than what was required. What is the ratio of nuts brought and to be brought?  Suppose he buys 24 nuts instead of 48. his son takes away 12 and his daughter takes away 6 out of the remaining. He finally has 42 less than required. Thus original requirements : 70/42* 48 = 80  thus ratio is : 1:8 answer  (you may assume any numbers but answer will be same).
A,B,C each deposit Rs. 1000 in a partnership. Out of this A takes away half of his capital after 6 months, B takes away 25% of capital 4 months before the year end. And C doubled his money after 3 months. Out of profit of 82000 how much will A get?  A _ (6*1000) + (6*500) = 9000 B= (8*1000) + (750*4) = 11000 c = (3*1000) + (9*2000) = 21000 their profit sharing ratio : 9:11:21 A will get : 9/(9+11+21)  * 82000 = 18000 answer.
Who is Noble peace prize winner for 2009?  Barak Obama
Who won booker award in 2009?  Hilary Mantel
Who is the president of United Nations ?  Dr. Ali Abdussalam Treki
How many seats did congress win in the last national election?  206 out of UPA 259 seats out of total 543
Who won the filmfare award for best film in 2008?  53 rd  filmfare award for best movie : tare zemeen pe
Who won the US open (female) in Tennis?  Justine Henin
Name the  books written by Chetan Bhagat ?  5 point someone  one night at call centre  3 mistakes of my life  2 states – a story of my marriage
Name the MD  of IMF?  Dominique Strauss-Kahn
WHO IS GOVERNOR OF RBI ?  Dr. D. Subbarao
WHO IS THE DIRECTOR OF 3 IDIOTS?  RAJKUMAR HIRANI
WHO WON THE DATASAHEB PHALKE AWARD 2008?  cinematographer V K Murthy
NAME AUTOBIOGRAPHIES OF INDIAN CRICKETER GAVASKAR  ?  Sunny Days : Sunil Gavaskar
HOW MANY CENTURIES HAVE BEEN HIT BY TENDULKAR ?  45 IN ONE DAY  46 TEST MATCHES  (46TH AGAINST BANGLADESH ON 21 DECEMBER 2009)
Which of these is correct ?  a. Having completed his lunch, a dog bit Mohan, who went to market. b. Having completed his lunch, Mohan went to market,  where he was bite by a dog.  c. Having completed his lunch, Mohan went to market, where he was bitten by a dog.  Answer : C.
Which of these is correct?  1. although he worked very hard, but he has failed.  2. although he had worked hard, yet he had been failed. 3. although he worked very hard, yet he failed.  Answer : 3
Which of these is correct?  1. He reached station before the train had reached  2. he had reached the station, where train had reached after him.  3. he had reached the station before the train.  Answer : 3.
Fill in the blank :  I prefer tea  _____ coffee options : from, then, than, to, on  ANSWER : TO
Arrange the following in proper order :  ,[object Object],[object Object],[object Object],[object Object],[object Object],[object Object]
Rearrange the following to make a meaningful paragraph:  ,[object Object],[object Object],[object Object],[object Object],[object Object],[object Object]
Ram and Shyam had equal money, which they deposited in two different banks. One got 10% simple interest, but the other got 5% half yearly compounding interest. At the end of 2 years, their amounts had difference of Rs. 10, what was the original amount deposited by them?  Let us suppose they deposited 100 each. First person will get : 100+10+10 = 120  2 nd  person will get : 100 + 5 + 5.25 + 5.5125 + 5.79 or 5.80 =  121.56  the difference is 1.56, if difference is 10, the principal should be : 10/1.56* 100 = 600 by each approx.
A shopkeeper uses weight of 900 grams instead of 1000 grams (1kg) while selling. He sells at his cost + 10%, what is his actual profit in %?  He sells 900 grams when he is charging for 1000 grams. Suppose he purchased 900 for Rs. 900. His cost is 900, but he is charging for 1000 + 10% = 1100 his profit is 200 on 900, or 200/900 *1000 = 22.2% answer
A circle (of maximum size possible) is inserted inside a squre. How much of the area of square is still left out of total area in %?  Let us suppose that the side of square is 14. area of square is 14*14 = 196  area of circle : 22/7*7*7 = 154  area left : 196-154 = 42 in % = 42/196 *100 = 21% approx. Answer you can assume any value instead of 14 and try to work out the %, the answer will be the same.
Out of the following chart, please, tell how many female students got between 50 - 60% in PGPSE ?
Solution... We can see that 12 girls got more than 60% marks.  40 – 12 – 2 = 26 got between 50 to 60%. answer
Based on the data given below, find out sale of Raju TV store. 1990 was the first year, when Raju TV sold 100 TV. Every person who buys TV buys again after 4 years. Raju sells 400, 600, 800 TV in 94, 98, 99 respectively. There was no new customer in 2000, but the sale was simple average of sale of 1990 and 1994.  How many TV did Raju TV sold in 1996.
solution.... Average of 1990 and 1994 = 250 no new customer in 2000, so 250 TV will be sold as repeat customers.  Thus sale in 1996 = 250 TV.
Ram, Ajay and Ravi (3 friends) are experts in three games – cricket, hockey and golf. One of these live in Delhi, and other two live in Mumbai. The person who lives in Delhi doesnt like Golf.  Hockey player lives in Mumbai.  The person whose name is smallest is also the best player of Golf.  Ajay daily goes to his  friend's home. Ravi has to visit Mumbai twice a year. Who plays cricket?  Ajay lives in Mumbai (he cant be Delhi, as he daily visits his friend). The person who lives in Delhi doesnt like Golf, he also cant play Hockey (Hockey player is in Mumbai), so that persn plays Cricket. So the person who lives in Delhi plays criciket. Ravi visits Mumbai,so he lives in Delhi. Ravi plays Cricket.
Nanu's maternal grand father, and maternal grand mother both are doctors. His father is Engineer, but his mother is Professor. A is elder to C, but Younger to B. D is wife of A. E is husband of B. The eldest person is a male but he is neither A nor Nanu. Out of two ladies, the elder is Doctor and younger is professor. Who is Engineer. Which statement is redudant?  A is engineer (as he is the second male in hierarchy). The last statement : out of two ladies, the elder is doctor and younger is professor, is redudant – as it is not required).
118, 66,  252, 137 .... Look at the series, you find that 66 has been computed from 118 as under :  1^3+1^3+8^2 = 66, similarly  6^3 + 6^2 = 252  so : 137 = 1^3 + 3^3 +7^2 = 77 answer
What will come next  :;;; 1,8,27,64... so on??? 1^3 = 1, 2^3 = 8, 3^3 = 27, 4^3 = 64, 5^3 = 125,  thus answer is 125.
A went to buy 2 shirts and 1 trousers. He carried Rs. 1000 with him (just sufficient) . The ratio of prices of shirt and trouser is : 7:6. however, he purchased 2 trousers and 1 shirt and returned back with Rs. 50 also. What will he pay, if he buys 2 shirts and 3 trousers?  2 Shirt + 1 Trousers = 1000 1 shirt + 2 trousers = 950  1 shirt – 1 trousers = 50, Now  from 1 st  equation: 2 trousers + 1 trousers + Rs. 100 = 900 thus 1 trousers = 300 and 1 shirt = 350.  he will spend 700+900 = 1600 answer.
X has to organise his lecture plan. He has to teach entrepreneurship, Planning, Finance, and Law on 4 days and one day is for IT. IT must be before Law, but Planning should not be on the 1 st  day. Finance and IT must be on any of the last 3 days. Prepare complete schedule.  Solution: Last day : law, 2 nd  last IT, 3 rd  last : Finance, 4 th  last : Plannning;  the first day : Entrepreneurship. It is better to solve the last day first in this types of questions.  Answer.
A,B,C,D are working on a project with different names. W is not A, B is not Y or Z. D and E have no relation to C and W Who is A?  Answer cannot be determined / data inadequate
There are two numbers whose HCF and LCM are 8 and 25, what are those numbers?  No such numbers are possible as LCM 25 is not divisible by 8, LCM is always divisible by HCF. Answer
Mayank Bothra wants to bring tiles for his room. His room's size is 28 feet by 20 feet. What should the size of square tile, so that minimum number of tiles are used?  Find HCF of 28 and 20. The HCF is 4, so the size of tile should be 4 feet by 4 feet. The total number of tiles required will be : (28*20)/(4*4)  = 35  tiles answer
A monkey is trying to climb a Date tree. He climbs 4 feet in 2 minutes, but in the next 1 minute, he slips by 3 feet. When will he reach at the top of this Date tree which is 36 feet high?  Actually he travels 1 feet in 3 minutes. He has to travel (36-4) = 32 feet, which will take 32*3 = 96 minutes. Then in the next 2 minutes he will be at the top of the tree. Thus he will take 98 minutes in all.
Mayank Bothra buys apples @ 4 for Rs. 3 and sells them @ 3 for Rs. 4. what is his profitability?  There are two important numbers 3 and 4, so let us take LCM of 3 and 4, this is 12. let us assume Mayank got 12 apples. He had to pay 9 to buy 12 apples. Now he goes to sell them, he gets Rs. 16 when he sells them. His profit is (16-9) – 7, so his profitability is 7/9 *100 = 77.77% answer
Naresh Jain makes payment to a shopkeeper. By mistake the shopkeeper reveses the digits of the amount to be paid and takes Rs. 54 more than what was required. What was the actual payment due?  Hit and try for this type of question. Try for 39 and 93. 93-39 = 54. The shopkeeper has taken 93 instead of 39.  So the amount due was 39. answer
A and B run opposite to an elevator. The ratio of the speed of elevator, A and B is : 1:6:3 Elevator is going up, but A and B are coming down. A takes 1 minute and B takes 2.5 minutes to come down. How much time will they take if elevator was not moving?  If elevator is not moving, they will take less time. Look at the speed ratios. A will take 1/6 time less – so he will take 60 seconds – 10 seconds= 50 seconds and B will take 1/3 time less, so he will take : 150 seconds – (150*1/3)  = = 100 seconds. Answer. 2.5 minutes = 150 seconds
Ajay has to pay to Mayank Rs. 1100 after 1 year. What should he pay now and settle the accounts? Rate of interest is 10% We have to find the present value of 1100 formula of present value is (Amount * 100)/ (100 + rate of interest * no. Of years)  = (1100 * 100) / (100 +100) = 1000
A, B,C, & D start a game. In the beginning all of them had equal amount. The winner has to collect half the amount of all other players. In the first round C wins. In the 2 nd  round B wins and in the 3 rd  round B again wins. At this time, their scores are : A : 50, B : 1250,  C : 250 D : ? What was the score of each of these in the beginning ?  The score of D should be equal to that of A = 50. Now add the score of all of these, we get 1600. Thus each of these should have score of 1600/4 = 400 answer
A thief steels a necklace and runs @ 10 Km  per hour at 3 pm. After 18 minutes,  a policeman starts chasing him @ 15 km per hour. Whenwill the thief be caught?  At the speed of 10, the thief would have run 10 *18/60 = 3 Km. So policeman has to chase him for this distance to start :  3 / (15-10) * 1/60  =
Mayank Bothra wants to get his room coloured. His room is 10 Ft by 20 Ft. Assuming that colouring costs Rs. 100 per sq. ft., how much will Mayank pay (for four walls), height of walls is 10 feet?  The side towards length :  20*10 = 200 the side towards width :  10*10 = 100 total 4 walls (200*2+100*2 )=600 ft.  Total payment : 600*100=Rs. 60000 ans.
Rishi has put his icecream in a rectangular bowl of 10*20 inch with depth of 5 inch. Now he wants to make a round ball out of this. What will be the diameter of this ball?  Total quantity of icecream = 10*20*5 = 1000 cubic inch volume of sphere : = 4/3*pi*radius^3 = 4/3  22/7*r*r*r = 1000 r= cubic root of (21000/88) r= 6.5 approx. Or diameter= 13 inch approx. Answer
In a typcial coding exercise, a person says that RAM= 32, and SITA= 49, what will be KRISHNA In that code?  RAM = R= 18, A=1, M = 13 so adding these RAM = 32 SITA, S=19, I=9, T=20,A=1 Adding these we get 49, so KRISHNA should be : K=11,R=18,I=9,S=19,H=8,N=14,A=1 TOTAL = 80 ANSWER
If Earth:Moon, and Sun:mercury,  what should be X: Titan  The Moon rotates round the Earth, Mercury rotates round the Sun, Titan rotates round the Saturn. So X = Saturn.
Rishi has a white cube of 5 inch dimensions. He cuts it to make smaller cubes of 1 inch each. Before this he dips the cube to colour one of its dimension as red. Now how many cubes will have red as a colour on one side?  The cube is cut into dimension of 1 inch each. Thus (5*5*5)/(1*1*1) = 125  thus we have now total 125 small cubes. Out of these 125, 25 cubes are such which have one dimension as red.
A group leader puts his group in rows and columns. He finds that the number of rows and columns are equal. He finds that 20 more persons have to come. If he puts these 20 persons, there is one more complete row or one more column. What is the number of persons in his group?  It is clear that there are 20 rows and 20 columns, so total number of persons is 400. add 20 more persons to it, you get :  20*20 = 400 + 20 = 420 answer
Mayank has some money in his pocket. He gives me half of this. Now he spends 75% of the remaining on purchasing a shoe. He then realises that he has to buy a T shirt also. So he spends 80% of the remaining on a T-shirt. He is now thirsty and so spends 20% on a mineral water bottle. He is left with Rs. 40. How much did he have in the beginning?  Start from last, multiply each value by 100/ (100-what % is spent) :  40*100/80 = 50 50*100/20 = 250 250*100/25 = 1000 1000*100/50 = 2000 thus he had 2000 in the beginning
Mayank Bothra has prepared a software, which puts the letters in alphabetic order, but one at a time in 1 second. How much time will the software take to put  T I G E R in ascending  or descending  order?  T I G E R  first of all E will be put in the beginning then G will be put after E. Then I will be put after G and then R will be put before T.  Total time : 4 seconds.  In descending order : 1 second only (as only R has to be put after T)
What is BODMAS?  B= BRACKET O = OF D=DIVISION M=MULTIPLY A = ADDITION  S=SUBTRACT apply these in this sequence only.  So 9+9/9-2*1of (2-3*1/3) = 9+1-2= 8
Which is bigger :  12/13  14/16  16/19  23/29  let us compare 12/13 and 14/16, logically 12/13 should be bigger (gap between 13 and 12 is only 1). still : cross multiply : 12*16 and 13*14 , which is bigger? = 12*16 is bigger so 12/13 is bigger. Similarly we have to compare all the values and find the biggest number.
What will come next in the series :  ANW, DKT, GFQ, ....?  The series is : A – D – G so next digit should be : J (gap of + 2) next series is : N-K-F so next : Y (increasing gap -2,-4,-6)  next series : W-T-Q = N (gap of -2) so answer : JYN answer
A goes from X to Y at 10 am and will reach at 12 and B goes from Y to X at 9 and will reach at 12. when will they cross each other?  A travels for 2hours, B for 3 hours. Let us assume that the distance is (2*3) = 6 KM. A travels @ 3 KM per hour and B @ 2 km per hour. At 9 B starts, till 10 he covers 2 KM. Now gap between A and B is 4 Km. Divide 4 by (2+3) = 4/5 = .8 , so they will meet at 10 + (.8 * 60) = 48 minutes. So They wil meet at 10 + 48 min. Ans
A puts an item at 10% discount, before doing this he had actually increased the marked price by 10%, what is the change in his revenue?  Let us assume that the earlier marked price is 100. add 10% = 110, now A reduces it by 10% (10% discount on marked price) so he gets 110-11 = 99 thus there is 1% reduction in his revenue or (10% * 10% = 1%)
A takes up 2 types of rice – one at 10 per kg and another at 50 per kg, how should he mix these to get a profit of 20% when he sells them @ 30 per KG?   Sale price is 30, so cost should be 30*100/120 = 25 in order to get the ratio of mixture, let us deduct 25 from 10 and 50, we get : 15 and 25, now reverse these ratio : 25:15 or 5:3,  thus rice of the rate 10 should be 5/(5+3) = 5/8 and rice of rate 50 should be 3/8  or their ratio should be 5:3 answer.
A and B are partners and share profit in the capital ratio of 3:2, however, A withdraws his half of the money after 5 months, and B doubles his money after 8 months, what is the final profit sharing ratio?  For first six months, the capitals are in the ratio of 3:2, but now there is some change.  A withdraws 1.5 and after 2 more months, B adds 2 more.  Thus (3*6months + 1.5*6months) : (2*8 months + 4*4months)  27: 32 answer
A:B = 2:3, C:D = 3:4, B:D = 3:2, what is A:C?  In A:B and B:D, we have B common, so  A:B:D = 2:3:2 We know that C:D is 3:4, so keep the value of D as 4, thus new ratio is :  A:B:C:D = 4:6:3:4 thus A:C = 4:3 answer
A can complete a work in 20 days and B can do the same work in 30 days. Both work together for 1 day and get 500 as total amount, how should this money be divided between A and B?  A: B should be 30:20 (reverse of 20:30 – the number of days taken by them in doing the work).  Thus A should get : 300 and B 200 answer.
A,B,C start a work together. A can do it alone in 20, B in 30 and C in 40 days, in how many days will they finish the work?  A's one day work : 1/20, B's one day's work : 1/30 and C's one day's work : 1/40,  add them : 1/20 + 1/30 + 1/40  (6+4+3)/120 = 13/120 , now reverse it :  120/13 = 9.2 days or 10 days approx.
There are 720 students in an examination, out of them, 1/3 rd  fail in English, 1/4 th  fail in Mathematics, 1/8 th  fail in both the paper. How many dont fail in any paper?  Out of 720, 240 fail in English, and 180 fail in Mathematics and 90 fail in both the paper. Thus the total number of students who have failed in either paper : 240+180 – 90 = 330 thus the students who have not failed in any paper is : 720 – 330 = 390 answer
Kapil is 4 th  from top and 106 th  from bottom. How many students are there in total ?  There are three students before Kapil, 105 below him and he himself  so : 3+105+1 = 109  or  4+106 – 1 = 109 answer
A and B have salary in the ratio of 10:11, but their savings are in the ratio : 3:7, but now their salaries increase by 100% each. If earlier their total savings was Rs. 2100, what is their new savings, if they dont change their expenditure and they spend equal amounts?  Their total savings : 2100, so A saves 3/10 *2100 = 630 and B saves : 7/10 * 2100 = 1470 difference of their savings is : 840, so A should earn 840 *10 = 8400 and spend (8400-630) = 7770 and B should earn 9240 and save (9240-1470) = 7770. thus their new incomes are : 16800 and 18480 and new savings are : (16800 – 7770) = 9030 and (18480-7770) = 10710 answer
A and B run a race and A gives B a start of 100 meters in 1000 meter race, still A wins by 1 second or 1 meters. What is the ratio of their speeds?  B takes 1 second to cover 1 meters, so his speed is 3600 meters per KM or 3.6 km per hour. He has actually run 900 meters only. He should take 900/3600*60 = 15 minutes A has run 1000 meters in 14 minutes and 59 seconds. So his speed is 40 KM per hour approx.  Thus the ratio of their speed is 40:36 or 10:9 answer
What is 10% of 30% of 1500?  30% of 1500 = 450  now 10% of 450 = 45 so answer is 45
Which is more 3^1/3 or 4^1/4?  Let us take the two powers : 1/3 and 1/4, let us now get same powers. LCM of 3,4 is 12, so let us multiply them by 4/4 and 3/3, we get : 4/12 and 3/12. we can now solve the numerator powers : so answer is : 81^1/12 and 64^1/12 now we can see that 81^1/12 is bigger, so the answer is 3^1/3 is bigger than 4^1/4
Ravi sees a photo and says that “this is the photo of younger son of the brother of the grandson of the father of the brother of the mother of my sister”, whose photo was it?  Answer : This is the photo of nephew of Ravi (son of Ravi's cousin) answer In order to solve such questions, start from last, and try to solve each relationship in simple terms or prepare a tree diagram from these statements. For example, : brother of the mother of my sister = maternal uncle – so similarly replace all these sentences.
MORE LINKS TO DOWNLOAD PRACTICE MATERIAL:  http://www.scribd.com/doc/23407950/mathematics-for-OPENMAT-MAT-CAT-GMAT
ADDITIONAL LINKS  http://www.scribd.com/doc/11692763/Advanced-Mathematics-for-GMAT-CAT-MAT http://www.scribd.com/doc/23407929/Advanced-Mathematics-for-GMAT-CAT-MAT http://www.scribd.com/doc/11625819/Basic-Mathematics-for-Cat-GMAT-Mat http://www.scribd.com/doc/23407954/mathematics-for-OPENMAT-MAT-CAT-GMAT-25-april http://www.scribd.com/doc/23407934/Advanced-Mathematics-for-GMAT-CAT-MAT2 http://www.scribd.com/doc/6583520/English-Afterschoool-21-May-2 http://www.scribd.com/doc/23300964/Advanced-Mathematics-for-GMAT-CAT-MAT2 http://www.scribd.com/doc/23407945/MATHEMATICS-FOR-ATMA-CAT-MAT-GMAT-BANK-PO-GRE
http://www.scribd.com/doc/6583303/Reasoning-Afterschoool
http://www.scribd.com/doc/23393680/REASONING-AFTERSCHOOOL http://www.scribd.com/doc/23393719/reasonning-44 http://www.scribd.com/doc/6583347/DI-and-Reasoning http://www.scribd.com/doc/23407636/10-July-reasoning http://www.scribd.com/doc/6583273/Reasoning-Quiz http://www.scribd.com/doc/23393476/10-July-reasoning http://www.scribd.com/doc/23393716/REASONING-QUIZ http://www.scribd.com/doc/14705025/17-Reasoning-II http://www.scribd.com/doc/23393478/10-July-reasoning-II
http://www.scribd.com/doc/23393316/general-knowledge http://www.scribd.com/doc/23609752/Group-Discussion-Afterschoool http://www.scribd.com/doc/6583547/General-Knowledge-24-May
THANKS.... GIVE YOUR SUGGESTIONS AND JOIN AFTERSCHOOOL NETWORK / START AFTERSCHOOOL SOCIAL ENTREPRENEURSHIP  NETWORK IN YOUR CITY  [email_address] PGPSE – WORLD'S MOST COMPREHENSIVE PROGRAMME IN SOCIAL ENTREPRENEURSHIP

Contenu connexe

En vedette

I N D U S T R I A L D E V E L O P M E N T & R E G U L A T I O N A C T & O...
I N D U S T R I A L  D E V E L O P M E N T &  R E G U L A T I O N  A C T &  O...I N D U S T R I A L  D E V E L O P M E N T &  R E G U L A T I O N  A C T &  O...
I N D U S T R I A L D E V E L O P M E N T & R E G U L A T I O N A C T & O...Dr. Trilok Kumar Jain
 
Today’s nation builder – suresh gyan vihar university where the students ...
Today’s  nation builder – suresh gyan vihar university    where the students ...Today’s  nation builder – suresh gyan vihar university    where the students ...
Today’s nation builder – suresh gyan vihar university where the students ...Dr. Trilok Kumar Jain
 
` H O W T O W I N D U P A S I C K C O M P A N Y
`  H O W  T O  W I N D  U P  A  S I C K  C O M P A N Y`  H O W  T O  W I N D  U P  A  S I C K  C O M P A N Y
` H O W T O W I N D U P A S I C K C O M P A N YDr. Trilok Kumar Jain
 
Article of professor trilok kumar jain in magazine proficient journal of mana...
Article of professor trilok kumar jain in magazine proficient journal of mana...Article of professor trilok kumar jain in magazine proficient journal of mana...
Article of professor trilok kumar jain in magazine proficient journal of mana...Dr. Trilok Kumar Jain
 

En vedette (7)

Abb annual report
Abb annual reportAbb annual report
Abb annual report
 
Accounting For A Partnership Firm
Accounting For A Partnership FirmAccounting For A Partnership Firm
Accounting For A Partnership Firm
 
I N D U S T R I A L D E V E L O P M E N T & R E G U L A T I O N A C T & O...
I N D U S T R I A L  D E V E L O P M E N T &  R E G U L A T I O N  A C T &  O...I N D U S T R I A L  D E V E L O P M E N T &  R E G U L A T I O N  A C T &  O...
I N D U S T R I A L D E V E L O P M E N T & R E G U L A T I O N A C T & O...
 
Today’s nation builder – suresh gyan vihar university where the students ...
Today’s  nation builder – suresh gyan vihar university    where the students ...Today’s  nation builder – suresh gyan vihar university    where the students ...
Today’s nation builder – suresh gyan vihar university where the students ...
 
` H O W T O W I N D U P A S I C K C O M P A N Y
`  H O W  T O  W I N D  U P  A  S I C K  C O M P A N Y`  H O W  T O  W I N D  U P  A  S I C K  C O M P A N Y
` H O W T O W I N D U P A S I C K C O M P A N Y
 
Daimler benz annual report
Daimler benz annual reportDaimler benz annual report
Daimler benz annual report
 
Article of professor trilok kumar jain in magazine proficient journal of mana...
Article of professor trilok kumar jain in magazine proficient journal of mana...Article of professor trilok kumar jain in magazine proficient journal of mana...
Article of professor trilok kumar jain in magazine proficient journal of mana...
 

Similaire à Prevention Of Mismanagement In Companies

Statistics, Index Numbers, And Analysis For Business 18 October
Statistics, Index Numbers, And Analysis For Business 18 OctoberStatistics, Index Numbers, And Analysis For Business 18 October
Statistics, Index Numbers, And Analysis For Business 18 OctoberDr. Trilok Kumar Jain
 
Questions of boat and speed for aptitude tests
Questions of boat and speed for  aptitude testsQuestions of boat and speed for  aptitude tests
Questions of boat and speed for aptitude testsDr. Trilok Kumar Jain
 
Selective school mathematics sample test
Selective school mathematics sample testSelective school mathematics sample test
Selective school mathematics sample testNotes Edu
 
Sec. 8.3 proportions
Sec. 8.3 proportionsSec. 8.3 proportions
Sec. 8.3 proportionsbweldon
 
Applications of rational equations powerpoint
Applications of rational equations powerpointApplications of rational equations powerpoint
Applications of rational equations powerpointDawn Adams2
 
(7) Lesson 1.7 - Constant Rate of Change
(7) Lesson 1.7 - Constant Rate of Change(7) Lesson 1.7 - Constant Rate of Change
(7) Lesson 1.7 - Constant Rate of Changewzuri
 
Percentages, simple and compound interest; time, distance and speed
Percentages, simple and compound interest; time, distance and speedPercentages, simple and compound interest; time, distance and speed
Percentages, simple and compound interest; time, distance and speedcolegiolascumbres
 
ratios and proportions in mathematics grade 5.ppt
ratios and proportions in mathematics grade 5.pptratios and proportions in mathematics grade 5.ppt
ratios and proportions in mathematics grade 5.pptvicenteangelique19
 
Guide de problem solving avec solution - préparation entretiens conseil
Guide de problem solving  avec solution - préparation entretiens conseilGuide de problem solving  avec solution - préparation entretiens conseil
Guide de problem solving avec solution - préparation entretiens conseilJocelyne Giambiaggi
 

Similaire à Prevention Of Mismanagement In Companies (20)

Mathematical shortcuts- DOWNLOAD ENABLED
Mathematical shortcuts- DOWNLOAD ENABLEDMathematical shortcuts- DOWNLOAD ENABLED
Mathematical shortcuts- DOWNLOAD ENABLED
 
Management Aptitude Test 3 November
Management Aptitude Test 3 NovemberManagement Aptitude Test 3 November
Management Aptitude Test 3 November
 
Mathematical shortcuts
Mathematical shortcutsMathematical shortcuts
Mathematical shortcuts
 
Aptitude test papers
Aptitude test papers Aptitude test papers
Aptitude test papers
 
Aptitude test papers
Aptitude test papers Aptitude test papers
Aptitude test papers
 
Statistics, Index Numbers, And Analysis For Business 18 October
Statistics, Index Numbers, And Analysis For Business 18 OctoberStatistics, Index Numbers, And Analysis For Business 18 October
Statistics, Index Numbers, And Analysis For Business 18 October
 
Questions of boat and speed for aptitude tests
Questions of boat and speed for  aptitude testsQuestions of boat and speed for  aptitude tests
Questions of boat and speed for aptitude tests
 
Selective school mathematics sample test
Selective school mathematics sample testSelective school mathematics sample test
Selective school mathematics sample test
 
Sec. 8.3 proportions
Sec. 8.3 proportionsSec. 8.3 proportions
Sec. 8.3 proportions
 
Correlation18october
Correlation18octoberCorrelation18october
Correlation18october
 
Aptitude test papers
Aptitude test papers Aptitude test papers
Aptitude test papers
 
Applications of rational equations powerpoint
Applications of rational equations powerpointApplications of rational equations powerpoint
Applications of rational equations powerpoint
 
2 propotion
2 propotion2 propotion
2 propotion
 
Management Aptitude Test 11 Nov Ii
Management Aptitude Test 11  Nov IiManagement Aptitude Test 11  Nov Ii
Management Aptitude Test 11 Nov Ii
 
(7) Lesson 1.7 - Constant Rate of Change
(7) Lesson 1.7 - Constant Rate of Change(7) Lesson 1.7 - Constant Rate of Change
(7) Lesson 1.7 - Constant Rate of Change
 
Percentages, simple and compound interest; time, distance and speed
Percentages, simple and compound interest; time, distance and speedPercentages, simple and compound interest; time, distance and speed
Percentages, simple and compound interest; time, distance and speed
 
ratios and proportions in mathematics grade 5.ppt
ratios and proportions in mathematics grade 5.pptratios and proportions in mathematics grade 5.ppt
ratios and proportions in mathematics grade 5.ppt
 
Fifty questions with answers in aptitude
Fifty questions with answers in aptitudeFifty questions with answers in aptitude
Fifty questions with answers in aptitude
 
All bank written math solution 2015 16
All bank written math solution 2015 16All bank written math solution 2015 16
All bank written math solution 2015 16
 
Guide de problem solving avec solution - préparation entretiens conseil
Guide de problem solving  avec solution - préparation entretiens conseilGuide de problem solving  avec solution - préparation entretiens conseil
Guide de problem solving avec solution - préparation entretiens conseil
 

Plus de Dr. Trilok Kumar Jain

Covid 19 novel corona virus pandemic
Covid 19 novel corona virus pandemicCovid 19 novel corona virus pandemic
Covid 19 novel corona virus pandemicDr. Trilok Kumar Jain
 
Korona crisis and 20 lakh crore self reliance scheme
Korona crisis and 20 lakh crore self reliance schemeKorona crisis and 20 lakh crore self reliance scheme
Korona crisis and 20 lakh crore self reliance schemeDr. Trilok Kumar Jain
 
Article on respected parmanand joshi ji
Article on respected parmanand joshi jiArticle on respected parmanand joshi ji
Article on respected parmanand joshi jiDr. Trilok Kumar Jain
 
Jain teerthankar vardhman mahaveer and contemporary problems
Jain teerthankar vardhman mahaveer and contemporary problemsJain teerthankar vardhman mahaveer and contemporary problems
Jain teerthankar vardhman mahaveer and contemporary problemsDr. Trilok Kumar Jain
 
Critical analysis rajasthan government budget 2020
Critical analysis rajasthan government budget 2020Critical analysis rajasthan government budget 2020
Critical analysis rajasthan government budget 2020Dr. Trilok Kumar Jain
 
Learning languages in school education
Learning languages in school educationLearning languages in school education
Learning languages in school educationDr. Trilok Kumar Jain
 
Happiness class india school system (1)
Happiness class india school system (1)Happiness class india school system (1)
Happiness class india school system (1)Dr. Trilok Kumar Jain
 
Covid 19 novel corona virus pandemic
Covid 19 novel corona virus pandemicCovid 19 novel corona virus pandemic
Covid 19 novel corona virus pandemicDr. Trilok Kumar Jain
 
Higher education in india hindi article (1)
Higher education in india hindi article (1)Higher education in india hindi article (1)
Higher education in india hindi article (1)Dr. Trilok Kumar Jain
 
Republic day special hindi article (1)
Republic day special hindi article (1)Republic day special hindi article (1)
Republic day special hindi article (1)Dr. Trilok Kumar Jain
 

Plus de Dr. Trilok Kumar Jain (20)

Exam reform
Exam reformExam reform
Exam reform
 
Selfreliant india covid19
Selfreliant india covid19Selfreliant india covid19
Selfreliant india covid19
 
Self reliant india campaign
Self reliant india campaignSelf reliant india campaign
Self reliant india campaign
 
Korona virus tragedy
Korona virus tragedyKorona virus tragedy
Korona virus tragedy
 
Covid 19 novel corona virus pandemic
Covid 19 novel corona virus pandemicCovid 19 novel corona virus pandemic
Covid 19 novel corona virus pandemic
 
Korona crisis and 20 lakh crore self reliance scheme
Korona crisis and 20 lakh crore self reliance schemeKorona crisis and 20 lakh crore self reliance scheme
Korona crisis and 20 lakh crore self reliance scheme
 
Article on respected parmanand joshi ji
Article on respected parmanand joshi jiArticle on respected parmanand joshi ji
Article on respected parmanand joshi ji
 
Corona covid 19 lockdown request
Corona covid 19 lockdown requestCorona covid 19 lockdown request
Corona covid 19 lockdown request
 
Gram swarajya rural development
Gram swarajya rural developmentGram swarajya rural development
Gram swarajya rural development
 
Jain teerthankar vardhman mahaveer and contemporary problems
Jain teerthankar vardhman mahaveer and contemporary problemsJain teerthankar vardhman mahaveer and contemporary problems
Jain teerthankar vardhman mahaveer and contemporary problems
 
Critical analysis rajasthan government budget 2020
Critical analysis rajasthan government budget 2020Critical analysis rajasthan government budget 2020
Critical analysis rajasthan government budget 2020
 
Inspector raj in india
Inspector raj in indiaInspector raj in india
Inspector raj in india
 
Learning languages in school education
Learning languages in school educationLearning languages in school education
Learning languages in school education
 
Happiness class india school system (1)
Happiness class india school system (1)Happiness class india school system (1)
Happiness class india school system (1)
 
Korona virus tragedy
Korona virus tragedyKorona virus tragedy
Korona virus tragedy
 
Holi article trilok kumar jain
Holi article trilok kumar jainHoli article trilok kumar jain
Holi article trilok kumar jain
 
Covid 19 novel corona virus pandemic
Covid 19 novel corona virus pandemicCovid 19 novel corona virus pandemic
Covid 19 novel corona virus pandemic
 
Higher education in india hindi article (1)
Higher education in india hindi article (1)Higher education in india hindi article (1)
Higher education in india hindi article (1)
 
Republic day special hindi article (1)
Republic day special hindi article (1)Republic day special hindi article (1)
Republic day special hindi article (1)
 
Budget special article (1)
Budget special article (1)Budget special article (1)
Budget special article (1)
 

Dernier

What's New in Teams Calling, Meetings and Devices March 2024
What's New in Teams Calling, Meetings and Devices March 2024What's New in Teams Calling, Meetings and Devices March 2024
What's New in Teams Calling, Meetings and Devices March 2024Stephanie Beckett
 
Use of FIDO in the Payments and Identity Landscape: FIDO Paris Seminar.pptx
Use of FIDO in the Payments and Identity Landscape: FIDO Paris Seminar.pptxUse of FIDO in the Payments and Identity Landscape: FIDO Paris Seminar.pptx
Use of FIDO in the Payments and Identity Landscape: FIDO Paris Seminar.pptxLoriGlavin3
 
New from BookNet Canada for 2024: BNC CataList - Tech Forum 2024
New from BookNet Canada for 2024: BNC CataList - Tech Forum 2024New from BookNet Canada for 2024: BNC CataList - Tech Forum 2024
New from BookNet Canada for 2024: BNC CataList - Tech Forum 2024BookNet Canada
 
Nell’iperspazio con Rocket: il Framework Web di Rust!
Nell’iperspazio con Rocket: il Framework Web di Rust!Nell’iperspazio con Rocket: il Framework Web di Rust!
Nell’iperspazio con Rocket: il Framework Web di Rust!Commit University
 
Merck Moving Beyond Passwords: FIDO Paris Seminar.pptx
Merck Moving Beyond Passwords: FIDO Paris Seminar.pptxMerck Moving Beyond Passwords: FIDO Paris Seminar.pptx
Merck Moving Beyond Passwords: FIDO Paris Seminar.pptxLoriGlavin3
 
Dev Dives: Streamline document processing with UiPath Studio Web
Dev Dives: Streamline document processing with UiPath Studio WebDev Dives: Streamline document processing with UiPath Studio Web
Dev Dives: Streamline document processing with UiPath Studio WebUiPathCommunity
 
SIP trunking in Janus @ Kamailio World 2024
SIP trunking in Janus @ Kamailio World 2024SIP trunking in Janus @ Kamailio World 2024
SIP trunking in Janus @ Kamailio World 2024Lorenzo Miniero
 
How to write a Business Continuity Plan
How to write a Business Continuity PlanHow to write a Business Continuity Plan
How to write a Business Continuity PlanDatabarracks
 
SALESFORCE EDUCATION CLOUD | FEXLE SERVICES
SALESFORCE EDUCATION CLOUD | FEXLE SERVICESSALESFORCE EDUCATION CLOUD | FEXLE SERVICES
SALESFORCE EDUCATION CLOUD | FEXLE SERVICESmohitsingh558521
 
"Debugging python applications inside k8s environment", Andrii Soldatenko
"Debugging python applications inside k8s environment", Andrii Soldatenko"Debugging python applications inside k8s environment", Andrii Soldatenko
"Debugging python applications inside k8s environment", Andrii SoldatenkoFwdays
 
The State of Passkeys with FIDO Alliance.pptx
The State of Passkeys with FIDO Alliance.pptxThe State of Passkeys with FIDO Alliance.pptx
The State of Passkeys with FIDO Alliance.pptxLoriGlavin3
 
Visualising and forecasting stocks using Dash
Visualising and forecasting stocks using DashVisualising and forecasting stocks using Dash
Visualising and forecasting stocks using Dashnarutouzumaki53779
 
Passkey Providers and Enabling Portability: FIDO Paris Seminar.pptx
Passkey Providers and Enabling Portability: FIDO Paris Seminar.pptxPasskey Providers and Enabling Portability: FIDO Paris Seminar.pptx
Passkey Providers and Enabling Portability: FIDO Paris Seminar.pptxLoriGlavin3
 
Gen AI in Business - Global Trends Report 2024.pdf
Gen AI in Business - Global Trends Report 2024.pdfGen AI in Business - Global Trends Report 2024.pdf
Gen AI in Business - Global Trends Report 2024.pdfAddepto
 
A Journey Into the Emotions of Software Developers
A Journey Into the Emotions of Software DevelopersA Journey Into the Emotions of Software Developers
A Journey Into the Emotions of Software DevelopersNicole Novielli
 
New from BookNet Canada for 2024: Loan Stars - Tech Forum 2024
New from BookNet Canada for 2024: Loan Stars - Tech Forum 2024New from BookNet Canada for 2024: Loan Stars - Tech Forum 2024
New from BookNet Canada for 2024: Loan Stars - Tech Forum 2024BookNet Canada
 
The Fit for Passkeys for Employee and Consumer Sign-ins: FIDO Paris Seminar.pptx
The Fit for Passkeys for Employee and Consumer Sign-ins: FIDO Paris Seminar.pptxThe Fit for Passkeys for Employee and Consumer Sign-ins: FIDO Paris Seminar.pptx
The Fit for Passkeys for Employee and Consumer Sign-ins: FIDO Paris Seminar.pptxLoriGlavin3
 
Transcript: New from BookNet Canada for 2024: BNC CataList - Tech Forum 2024
Transcript: New from BookNet Canada for 2024: BNC CataList - Tech Forum 2024Transcript: New from BookNet Canada for 2024: BNC CataList - Tech Forum 2024
Transcript: New from BookNet Canada for 2024: BNC CataList - Tech Forum 2024BookNet Canada
 
DevEX - reference for building teams, processes, and platforms
DevEX - reference for building teams, processes, and platformsDevEX - reference for building teams, processes, and platforms
DevEX - reference for building teams, processes, and platformsSergiu Bodiu
 
Transcript: New from BookNet Canada for 2024: Loan Stars - Tech Forum 2024
Transcript: New from BookNet Canada for 2024: Loan Stars - Tech Forum 2024Transcript: New from BookNet Canada for 2024: Loan Stars - Tech Forum 2024
Transcript: New from BookNet Canada for 2024: Loan Stars - Tech Forum 2024BookNet Canada
 

Dernier (20)

What's New in Teams Calling, Meetings and Devices March 2024
What's New in Teams Calling, Meetings and Devices March 2024What's New in Teams Calling, Meetings and Devices March 2024
What's New in Teams Calling, Meetings and Devices March 2024
 
Use of FIDO in the Payments and Identity Landscape: FIDO Paris Seminar.pptx
Use of FIDO in the Payments and Identity Landscape: FIDO Paris Seminar.pptxUse of FIDO in the Payments and Identity Landscape: FIDO Paris Seminar.pptx
Use of FIDO in the Payments and Identity Landscape: FIDO Paris Seminar.pptx
 
New from BookNet Canada for 2024: BNC CataList - Tech Forum 2024
New from BookNet Canada for 2024: BNC CataList - Tech Forum 2024New from BookNet Canada for 2024: BNC CataList - Tech Forum 2024
New from BookNet Canada for 2024: BNC CataList - Tech Forum 2024
 
Nell’iperspazio con Rocket: il Framework Web di Rust!
Nell’iperspazio con Rocket: il Framework Web di Rust!Nell’iperspazio con Rocket: il Framework Web di Rust!
Nell’iperspazio con Rocket: il Framework Web di Rust!
 
Merck Moving Beyond Passwords: FIDO Paris Seminar.pptx
Merck Moving Beyond Passwords: FIDO Paris Seminar.pptxMerck Moving Beyond Passwords: FIDO Paris Seminar.pptx
Merck Moving Beyond Passwords: FIDO Paris Seminar.pptx
 
Dev Dives: Streamline document processing with UiPath Studio Web
Dev Dives: Streamline document processing with UiPath Studio WebDev Dives: Streamline document processing with UiPath Studio Web
Dev Dives: Streamline document processing with UiPath Studio Web
 
SIP trunking in Janus @ Kamailio World 2024
SIP trunking in Janus @ Kamailio World 2024SIP trunking in Janus @ Kamailio World 2024
SIP trunking in Janus @ Kamailio World 2024
 
How to write a Business Continuity Plan
How to write a Business Continuity PlanHow to write a Business Continuity Plan
How to write a Business Continuity Plan
 
SALESFORCE EDUCATION CLOUD | FEXLE SERVICES
SALESFORCE EDUCATION CLOUD | FEXLE SERVICESSALESFORCE EDUCATION CLOUD | FEXLE SERVICES
SALESFORCE EDUCATION CLOUD | FEXLE SERVICES
 
"Debugging python applications inside k8s environment", Andrii Soldatenko
"Debugging python applications inside k8s environment", Andrii Soldatenko"Debugging python applications inside k8s environment", Andrii Soldatenko
"Debugging python applications inside k8s environment", Andrii Soldatenko
 
The State of Passkeys with FIDO Alliance.pptx
The State of Passkeys with FIDO Alliance.pptxThe State of Passkeys with FIDO Alliance.pptx
The State of Passkeys with FIDO Alliance.pptx
 
Visualising and forecasting stocks using Dash
Visualising and forecasting stocks using DashVisualising and forecasting stocks using Dash
Visualising and forecasting stocks using Dash
 
Passkey Providers and Enabling Portability: FIDO Paris Seminar.pptx
Passkey Providers and Enabling Portability: FIDO Paris Seminar.pptxPasskey Providers and Enabling Portability: FIDO Paris Seminar.pptx
Passkey Providers and Enabling Portability: FIDO Paris Seminar.pptx
 
Gen AI in Business - Global Trends Report 2024.pdf
Gen AI in Business - Global Trends Report 2024.pdfGen AI in Business - Global Trends Report 2024.pdf
Gen AI in Business - Global Trends Report 2024.pdf
 
A Journey Into the Emotions of Software Developers
A Journey Into the Emotions of Software DevelopersA Journey Into the Emotions of Software Developers
A Journey Into the Emotions of Software Developers
 
New from BookNet Canada for 2024: Loan Stars - Tech Forum 2024
New from BookNet Canada for 2024: Loan Stars - Tech Forum 2024New from BookNet Canada for 2024: Loan Stars - Tech Forum 2024
New from BookNet Canada for 2024: Loan Stars - Tech Forum 2024
 
The Fit for Passkeys for Employee and Consumer Sign-ins: FIDO Paris Seminar.pptx
The Fit for Passkeys for Employee and Consumer Sign-ins: FIDO Paris Seminar.pptxThe Fit for Passkeys for Employee and Consumer Sign-ins: FIDO Paris Seminar.pptx
The Fit for Passkeys for Employee and Consumer Sign-ins: FIDO Paris Seminar.pptx
 
Transcript: New from BookNet Canada for 2024: BNC CataList - Tech Forum 2024
Transcript: New from BookNet Canada for 2024: BNC CataList - Tech Forum 2024Transcript: New from BookNet Canada for 2024: BNC CataList - Tech Forum 2024
Transcript: New from BookNet Canada for 2024: BNC CataList - Tech Forum 2024
 
DevEX - reference for building teams, processes, and platforms
DevEX - reference for building teams, processes, and platformsDevEX - reference for building teams, processes, and platforms
DevEX - reference for building teams, processes, and platforms
 
Transcript: New from BookNet Canada for 2024: Loan Stars - Tech Forum 2024
Transcript: New from BookNet Canada for 2024: Loan Stars - Tech Forum 2024Transcript: New from BookNet Canada for 2024: Loan Stars - Tech Forum 2024
Transcript: New from BookNet Canada for 2024: Loan Stars - Tech Forum 2024
 

Prevention Of Mismanagement In Companies

  • 1. MOCK MAT / RMAT / CET by : DR. T.K. JAIN AFTERSCHO ☺ OL centre for social entrepreneurship sivakamu veterinary hospital road bikaner 334001 rajasthan, india FOR – PGPSE PARTICIPANTS mobile : 91+9414430763
  • 2. Be Quicker faster accurate
  • 3. Download material http://www.4shared.com/file/129613569/c4e24e77/_2__MOCKCAT.html http://www.4shared.com/file/129077728/276ee61/CD_MOCK_MAT_QUIZ_MAY_2009.html http://www.4shared.com/file/129075830/b4e3d8a4/cdmat.html http://www.4shared.com/file/129078229/2bd90c4a/cd_write_mock_mat.html http://www.scribd.com/doc/27133626/Rmat-Mat-GMAT-Cat-Questions http://www.scribd.com/doc/23610014/Reasoning-e-Book http://www.4shared.com/file/129078352/f2882932/_2__cdmat.html
  • 4. QUESTIONS ON BASIC MATHEMATICS TURN TO NEXT SLIDES :....
  • 5. Tea worth Rs 120 & 130 per Kg are mixed with a third variety in the ratio 1:1:2. If the mixture is worth Rs 150 per Kg , the price of the third variety per Kg will be?
  • 6. SOLUTION Let us assume that we use 1kg of first two varieties and 2 kg of 3 rd variety. Total is 4 kg. So the total price is : 150*4 = 600 price paide for 1 st two varieties : (120+130) = 250 price paid for 3 rd variety : 600-250 = 350 Price per kg for 3 rd variety : 175 per kg.
  • 7. A bill for Rs.6000 is drawn on July 14 at 5 months . It is discounted on 5th October at 10%.Find the bankers discount true discount, bankers gain and the money that the holder of the bill receives. Due date of bill : 17 december interest for period (17 dec – 5 oct) = 73 days interst : 6000*73/365 * 10/100 = 120 this is banker's discount, the bank will give 5880 (money the holder will get ) formula for present value : 6000 * 100/ (100+rt) = 6000 *100/102 =5882.35 true discount is 117.65 answer
  • 8. In one hour a boat goes 11 km with the stream and 5 km against the stream. The speed of the boat in still water is? The speed in still water is simple average of the two speeds. So it is ½ ( 11+5) = 8 Km. So the speed of boat in still water is 8 km per hour.
  • 9. A boat can travel with a speed of 13 kmph in still water. if the speed of stream is 4 kmph,find the time taken by the boat to go 68 km downstream? In downstream the speed of boat and stream will add up. 13+4 = 17 km per hour. Thus 68/17 = 4 hours. The boat will take 4 ours to cover a distance of 68 km downstream. Answer
  • 10. A boat takes 19 hrs for travelling downstream from point A to point B. And coming back to a point C midway between A and B. if the velocity of the sream is 4 kmph . and the speed of the boat in still water is 14 kmph. what is the distence between A and B? The boat initially travelled at (14+4) = 18 km per hour. Then it travelled at (14-4) = 10 km per hour. Let us assume distance from A to B to be 2X. So it has travelled : 2x/18 + x/10 = 19 38X = 3420 x= 3420/38 = 90 so distance = 180 ans.
  • 11. A boat takes 90 min less to travel 36 miles downstream then to travel the same distence upstream. if the speed of the boat in still water is 10 mph . the speed of the stream is : 90 minute = 1.5 hours Let us assume the speed of stream = x 36/ (10-x) – 36/ (10+x) = 1.5 so X = 2.
  • 12. A can contains a mixture of two liquids A and B in the ratio 7:5 when 9 liters of mixture are drawn off and the can is filled with B,the ratio of A and B becomes 7:9. How many liters of liquid A was contained by the can initially? Total quantity is same before and after = X. The only change is in the ratio. 9 liters are taken out in ratio : 7:5, thus 9*7/12 = 5.25 is A and 3.75 is B. We can write it as : 5/12 X + (9-3.75) = 9/16 X 5/12X -9/16X = -5.25 -7/48X = -5.25 or X = 360 the original quantity of A = 360 * 7/12 = 210 answer
  • 13. .One quantity of wheat at Rs 9.30 per Kg are mixed with another quality at a certain rate in the ratio 8:7. If the mixture so formed be worth Rs 10 per Kg ,what is the rate per Kg of the second quality of wheat? Let us assume that the two types of wheat is mixed 8Kg and 7Kg respectively – total is 15 kg. Total cost : 15*10 = 150. Price paid for 1 st type : 9.3*8 = 74.4 price paid for 2 nd : 150 – 74.4 = 75.6 divide it by 7 to get price of 2 nd : 75.6 / 7 = 10.8 answer
  • 14. Tea worth Rs 120 and 130 per Kg are mixed with a third variety in the ratio 1:1:2. If the mixture is worth Rs 150 per Kg ,the price of the third variety per Kg will be? We have taken 1 kg of 1st, 1kg of 2 nd and 2kg of 3rd. Total = 4 kg. Thus total price = 150*4 = 600. price paid on 1 st and 2 nd = 250. price paid on 3 rd : 600-250 = 350 price per kg = 350/2 = 175 per kg. Answer
  • 15. A vessel is filled with liquid,3 parts of which are water and 5 parts syrup. How much of the mixture must be drawn off and replaced with water so that the mixture may be half water and half syrup? The ratio of water and syrup is 3:5. total is 8. we want them to be equal : 4:4. thus we want syrup to be 4. now syrup is 5. thus we have to take 1 out of 5. Thus the ratio in which mixture has to be drawn out is 1/5. thus 8 * 1/5 = 1.6 must be drawn out, which in 3:5 is : .6 and 1 thus after drawing, the water and syrup is 2.4 and 4. add 1.6 water again, and it is 4:4. so 1/5 th of the mixture should be drawn off. Answer.
  • 16. The price of an article including the sales tax is Rs 616.The rate of sales tax is 10%,if the shopkeeper has made a profit of 12%,then the cost price of the article is? Let us assume cost = 100 add profit(+12) = 112 add sales tax (11.2) = 123.2 thus 100/123.2 * 616 = cost = 500 answer
  • 17. By selling 33m of cloth ,one gains the selling price of 11m.Find the gain percent? Suppose sale price is 1 per meter. Total sale price = 33 profit = 11 cost = 33-11 = 22 thus profit is : 11/22 * 100 = 50% of cost. Answer
  • 18. Find 3 numbers in the ratio : 2:3:5, the sum of whose squares is 608? 2x^2 +3x^2 +5x^2 = 608 38x^2 = 608 X^2 = 608/38 = 16 x= 4 numbers are 8, 12, 20 ans.
  • 19. Download links ...for time and work problems http://www.scribd.com/doc/24305787/Time-amp-Work-amp-Time-amp-Distance-in-Quantitative-Aptitude
  • 20. Download links.. http://www.scribd.com/doc/6583802/1-July-Mathematics-Basics http://www.scribd.com/doc/11625711/Basic-Mathematics-1 http://www.scribd.com/doc/19628963/Mathematics http://www.scribd.com/doc/6683933/Probability-9-Sept http://www.scribd.com/doc/11626612/Probability http://www.scribd.com/doc/6683926/Probability-8-SEPT http://www.scribd.com/doc/25885859/Probability-for-Business-Management-Simplified http://www.scribd.com/doc/23301009/PROBABILITY http://www.scribd.com/doc/24894086/Basics-of-Permutation-and-Combination http://www.scribd.com/doc/7378688/6-August-Quantitative-Aptitude
  • 21. Download links .... http://www.scribd.com/doc/23407732/22-jULY-BUSINESS-MATHEMATICS http://www.scribd.com/doc/23407815/28-July-Business-Mathematics http://www.scribd.com/doc/23407867/29-JULY-QUANTITATIVE-APTITUDE http://www.scribd.com/doc/7378688/6-August-Quantitative-Aptitude http://www.scribd.com/doc/23393694/14-June-aptitude-test-II http://www.scribd.com/doc/6583737/18-JUNE-Comprehensive-Test
  • 22. Download links... http://www.scribd.com/doc/14675434/29-July-Quantitative-Aptitude http://www.scribd.com/doc/23393688/14-June-aptitude-test
  • 23. Difficult words ... Turn to next slides ...
  • 24. Download links http://www.scribd.com/doc/6583519/English-Afterschoool-21-May http://www.scribd.com/doc/6906830/word-power-afterschoool http://www.scribd.com/doc/6583315/English-Improvement-Afterschoool http://www.scribd.com/doc/6583303/Reasoning-Afterschoool http://www.scribd.com/doc/6583520/English-Afterschoool-21-May-2 http://www.scribd.com/doc/6583519/English-Afterschoool-21-May http://www.scribd.com/doc/14660441/English-Afterschoool-23-May http://www.scribd.com/doc/22088672/Basic-Mathematics http://www.scribd.com/doc/19828297/Mathematics
  • 25. Find the meaning of the following words ? 1.Sage : highly learned person 2.Salubrious : healthful 3.Sap : diminish , Undermine 4.Satiate : Satisfy fully 5.Savor : enjoy 6.Sear : char or burn , 7. Scad : great quantity 8.Spate: sudden flood 9.Sodden : Soaked , dull 10.Snivel : whine, to complain 11. Smirk : concited smile. 2.Slacken : slowup, loosen 13.Sineway: tough, setstrong and firm 14Shyster : lawyer using Questionable methods 15.Shard : fragment , generally of pottery 16.Stanch:Check flow of blood. 17.Stint : be thrifly, set limits 18.Stolid : dull , impassive 19. subside : settleddown, reduce 20.Sylvan: pertaining to the woods 21.Sybarite : lover of luxury. 22.Swindles : cheat
  • 26. Difficult words and their meanings ... 1.Rankle=irritate,fester 2.Rancid=having bad odor 3.Raspy=harsh 4.Revage=plunder,despoil 5.Ravenous=extremely hungry 6.Realm=area of work 7.Renege=deny 8.Repast=meal,feast 9.Regal=royal 10.Rig=fix,manipulate 11.Roster=list 12.Reek=emit(odor) 13.Reiterate=repeat 14.Remorse=guilt,self-reproach 15.Regicide=murder of a king or queen 16.Refectory=dining hall 17.Redoient=fragrant,odorous, 18.Retinue=following,attendants 19.Ruse=trick 20.Ruffian=bully,scoundrel
  • 27. How many times does the hands of a clock make right angle? Let us take from 6 am to 6 pm. At 6.15 it is almost right angle (not exact), similarly at about 6.45 also there is a right angle. Thus we have 2 occasions between 6 and 7. We should have 12 hours so 24 occasions when it can be in right angle. But between 9 to 10 we have only 1 such occasion (9.30), again from 3 to 4, we have only one such case. Thus in 12 hours we have only 22 right angles. Answer
  • 28. Why is it so? We can understand that by the time the minute hand covers 55 minutes, there is one hour (60 minutes). Thus there is a gap of 5 minutes. We can see that at times like 9 and 3 there is a problem. 9 am is counted in 8 to 9 (8.25 and 8.59) to count right angle. And so is the case with 3 pm). If it is asked what is the exact time of right angle, use 60/11 as a formula to find exact time.
  • 29. How many times do we have a straight angle between two hands of a clock from 6 am to 6 pm in a day? Ideally it should be 12 (but it will not be 12) . start at 6, 7.05, 8.10, 9.15, 10.20, 11.25, 12.30, 1.35, 2.40, 3.45, 4.50, 5.55, 6. thus we have 13 such occasions when we have straight angle.
  • 30. A watch which gains uniformly,is 5 min,slow at 8 o'clock in the morning on sunday and it is 5 min.48 sec.fast at 8 p.m on the following sunday. when was it correct? The watch has actually covered 10 min. 48 sec. In a span of 180 hours. We want to know about the exact time when it had crossed only 5 minutes. So solve it using following formula : 5/ 10.8 (180) = 82.8 thus exact time is : 82/24 = 3 day + 10 hours Wednesday : 6 pm + 48 minutes. Answer
  • 31. Explanation Actually 180 is hours (from 8am to 8pm on next Sunday). Actually 82.8 is in hours, to convert it into days, we have devided 82/24, we get 3 rd day + 10 hours. Why did we use : 10. 8 : note: we converted 10.48 in decimal : 48/60 = .8 so it was 10.8
  • 32. Questions on analogy ... Next slide .....
  • 33. Errata : Books : : flaws:? (A)Manuscripts (B)Metals (C)Speech (D)Charter Metals (these are errors in flaws just like errata (errors ) in books)
  • 34. Tractor : Trailer : : Horse : ? (A)Stable (B)Cart (C)Saddle (D)Engine Cart
  • 35. Eye :Myopia : : Teeth : ? (A)Pyorrhea (B)Cataract (C)Trachoma (D)Eczema Pyorrhea (name of disease)
  • 36. Fossils : Creatures : : Mummies : ? (A)Egypt (B)Human beings (C)Animals (D)Martyrs Human beings
  • 37. Reluctant : Keen : : Remarkable : ? (A)Usual (B)Restrained (C)Striking (D)Evolution Reluctant = not interested keen = interested (so they are opposite) the opposite of remarkable is usual
  • 38. Sculptor : Statue : : Poet : ? (A)Canvas (B)Pen (C)Verse (D)Chisel Sculptor makes statue Poet makes Verse
  • 39. Pesticide : Crop : :Antiseptic : ? (A)Wound (B)Clothing (C)Bandage (D)Bleeding Wound pesticide is used to destroy antibodies in crop, the same role is played by antiseptic on wound
  • 40. Engineer : Map : : Bricklayer : ? (A)Design (B)Template (C)Mould (D)Cement Template
  • 41. Fire : Ashes : : Explosion : ? (A)Flame (B)Death (C)Sound (D)Debris Debris
  • 42. Quail : Partridges : : Yak : ? (A)Cows (B)Deer (C)Oxen (D)Antelope Similar to them is the relation between Yak and Oxen
  • 43. Bank : River : : Coast : ? (A)Flood (B)Waves (C)Sea (D)Beach Sea (Coast is the border of Sea, just like a bank is the border of river)
  • 44. What is the opposite of the following words Jaded (to mock / discourage) : Motivate / stimulate .Jaundiced (biased) = Unbiased .Jaunty (unbalanced, excited) = sedate (calm) Jeopardy (to put in danger) = Safety Jettison (to throw out of ship) x Salvage (to save)
  • 45. Antonyms : LACHRYMOSE (which produces tears) = CHEERING (removes tears) LACKADAISICAL (lazy)= AMBITIOUS LACONIC (in a few words) = VERBOSE (in lot of words) LAMPOON (to ridicule) =PRAISE LANGUOR (in depression) = VITALITY (in youth and zeal) LAVISH (who spends a lot) = FRUGAL (spending less) .LAUDATORY (remarkable) = DEFAMATORY (infamous) LECHERY (without character)= PURITY LETHARGIC (lazy)= INVIGORATING (full of zeal ) LEVITY (lightly)= SOLEMNITY (serious) LIMPID (clear) X TURBID (dirty) LOATH (to hate)= EAGER (keen) .LOQUACIOUS (talkative) = TACITURN (laconic) LUGUBRIOUS (mournful, sad) =CHEERFUL .LURID (colourful) = DULL
  • 46. ANTONYMS MALIGAN(to criticise) – Eulogize / praise MALLEABLE (ductile) = Brittle .MANIACLE (handcuff, chained) = Sane MANIFEST (visible)= Obscure MANUMIT (to free) = Enslave
  • 47. Cobbler : Leather : : Carpenter : ? (A)Furniture (B)Wood (C)Hammer (D)Chair Wood
  • 48. Rupee : Indian : : Yen : ? Japan
  • 49. Ocean : Deserts : : Waves : ? (A)Sea (B)Dust (C)Sand dunes (D)Ripples Sand Dunes
  • 50. Illiteracy : Education : : Flood : ? (A)Rain (B)Bridge (C)Dam (D)River Dam
  • 51. Appraiser : Building : : Critic : ? (A)Book (B)Masterpiece (C)Judge (D)Gold Book
  • 52. Dog : Bark : : Goat : ? (A)Bleat (B)Howl (C)Grunt (D)Bray Bleat
  • 53. Oxygen : Burn : : Carbon dioxide : ? (A)Isolate (B)Foam (C)Extinguish (D)Explode Extinguish
  • 54. Water : Convection : : Space : ? (A)Conduction (B)Transference (C)Vacuum (D)Radiation Radiation
  • 55. Venerate : Worship : : Extol : ? (A)Glorify (B)Homage (C)Compliment (D)Recommend Venerate = worship = to pray similarly, extol = to praise = Glorify
  • 56. Data sufficiency question... Next. ... slide ...
  • 57. How is M related to N ? 1. P who has only 2 kids: M& N. P is the mother in law of Q, who is sister in law of N 2. R, the sister in law of M, is the daughter in law of S, who has only 2 kids – M and N. From the first statement it is clear that M is Man. (M is husband of Q) from second statement it is clear than N is also Man (N is husband of R). thus M is brother of N But for getting the answer, first statement is sufficient, so take care in exam, write only 1 st statement is sufficient to answer.
  • 58. How is T related to the man in photograph? a. The man in photograph is the only son of T's grandfather b. The man in the photo has no brother / sister and his father is T's Grandfather. The answer can be drawn from either statement (alone). Thus we can use any of these statements to answer the question.
  • 59. two whole numbers whose sum is 64 cant be in the ratio ? Options : 3:5, 1:7 3:4 9:7 answer : as you can see that the sum of 3 and 4 is 7, which cant divide 64, so this is the answer.
  • 60. If a carton containing dozen mirrors is dropped, which of the following cant be the ratio of broken mirrors to unbroken mirrors? Options : 2:1 3:1 3:2 7:5 again 3+2 is 5, which cant divide 12 so this cant be the ratio of broken to unbroken mirrors. So answer is 3:2
  • 61. A father's age was 5 times son's age Five years ago. It will be 3 times after 2 years. What is the ratio of their present age? Father = X, Son = Y Now (X-5) = 5 (Y-5) (from first statement) (X+2)=3(Y+2) solving these two equations, we get : X-5Y=-20 and X-3Y=4 , so Y = 12 X=40 thus their ratio is : 10:3 answer
  • 62. Vinay got thrice as many marks in maths as in English. The proportion of marks in Maths and History is 4:3. If total marks in Maths, english, and History are 250, what are his marks in English? M: E = 3:1, M:H = 4:3, taking M as common and using cross multiplication, M : E : H : 12: 4:9 total = 25 M = 12/25 * 250 = 120 E = 4/25 * 250 = 40 H = 9/25*250 = 90 answer
  • 63. Data sufficiency question : out of which of the two option, can you get the answer ? Question : 1. What is the rank of Mayank in the class ? Options : a. Swati is 5 th from the bottom, and she is 30 places behind Mayank b. There are 40 students after Mayank.
  • 64. Solution From the first statement, or from second statement, we can get the rank from bottom, but not from top. We dont know how many students are there in the class. So we are NOT able to get answer from any of these two statements. Answer
  • 65. Data sufficiency question : from which of the following can we draw the answer ? Question : What is the volume of the cubical pot? Options : 1. the depth of the pot is equal to its width and the diagonal of the cubical pot is more than double the width. 2. The size of maximum size of ball that can be put in this pot is almost 80% of volume of this pot.
  • 66. Solution Answer cannot be drawn. None of the statement gives some help in finding volume of the cubical pot.
  • 67. Data sufficiency question ? Question : How many Sons does X has ? Options : 1. In the garden A was shouting X as father 2. X says that he has 1 offspring
  • 68. Solution None of the statement is of any help
  • 69. Data sufficiency question Question : How many persons are sitting in a roundtable?. Options : 1. There are 3 persons to the right of X 2. Y has 2 persons to his left and 2 to his right.
  • 70. Solution Answer cannot be drawn from either statement.
  • 71. Data sufficiency question ? Q : which direction is Ankit facing ? Options : 1. The rays of Sun are falling to the Right of Ankit. 2. It is morning time now.
  • 72. Answer Combining both the options, we can draw the answer. Ankit is North facing as his shadow is toward left and rays of Sun are towards right. .
  • 73. Two equal glasses are respectively 1/3 and ¼ full of milk. They are then filled up with water and the contents mixed in a tumbler. The ratio of milk and water in the tumbler is ? Let us assume that both the glasses have (3*4) = 12 liters of capacity. The first has 4 liters of milk and 2 nd glass has 3 liters of milk. Combined together they have 7 liters of milk and 17 liters of water. Thus the ratio of milk and water is : 7:17 answer
  • 74. The incomes of A and B are in the ratio of 3:2, and their expenditures are in the ratio 5:3. if each saves Rs. 1000, what is the income of A? Incomes are in ratio of X and expenditures are in the ratio of Y. 3X-5Y = 1000 2X-3Y=1000 -Y=-1000, X = 2000 thus incomes of A and B : 6000 and 4000 and expenditures are : 5000 and 3000 answer.
  • 75. A sum of Rs. 1300 is divided between A,B,C,D such that A's share divided by B's share = B's share / C's share = C's share / D's share = 2/3 In such questions, start from Last – and take cube of the last number to start with – assuming that value for the last variable. (cube , if there are 3 iteration, if there are 4 iterations, take quadruple) Let us start from D, assume it to be 27, now C is 18 now accordingly : B is 12. now A is 8 the numbers are : 8, 12,18,27 thus A should get : 8/ 65 *1300 = 160
  • 76. Find the third proportion to 9 : 12? 9:12 :: 12:X multiply 9 by X and 12 to 12. 9X = 144 X = 16 answer
  • 77. Ankit looks at a photograph and says : “It is the photo of the only daughter of the maternal Grandfather of the only son of brother in law of only son of my maternal grand father.” whose photograph was it ? For this question, solve from last. Convert each clause into a single word and move forward. Only son of my maternal granfather = Mama (maternal uncle) Brother in law of Mama = Papa (father). So on.... you will find the photo is that of Ankit's Mother. Answer
  • 78. In a class out of 24 students 14 passed in Enlgish, 14 passed in Hindi, 12 passed in Both the papers. How many failed in both the papers? For such questions, prepare a venn diagram to solve it . Answer is 8. See it in next slide :
  • 79. Total = 24, out of box = 24-16 =8 = failed in both. 12 English Hindi 2 2 2
  • 80. A:B = 2:3, B:C=5:7, find A:B:C Here B is common. So cross multiply to solve it. Thus 5 must be multiplied to 2:3 and 3 must be multiplied to 5:7 answer : A:B:C= 10:15:21 answer
  • 81.  
  • 82. Mayank has 20 shares of RIL and market price of one share is 1060. How much money will he get by selling them. Brokerage rate is 1%? Sale price = 1060*20 = 21200 less 1% brokerage = 212 net amount = 20988 answer
  • 83. Atal does a work in 6 hours and Sonia does the same work in 9 hours. They work together. How much time will they take? 1/6 + 1/9 = (9+6) / 54 now multiply it by ½ ( as there are two persons here) : 15/54 * ½ = 15/108 now reverse it : 108/15 = 36/5
  • 84. A,B,C together complete a work in 6 days. A and B together do the same work in 10 days. How much time will C alone take to do the work? 1/6 – 1/10 = (10-6) / (10*6) = 4/60 = 1/15 reverse it : 15. so C alone will take 15 days to complete the work.
  • 85. Divide 27x^3 – 8 y^3 by 3x-2y (3x-2y)^3 = 27x^3 – 8 y^3 so remove its one power we get : (3x-2y )^2 we get : 9x^2-6xy+4y^2 answer
  • 86. Mayank shows a photograph and says that the mother of this person is the wife of the person who is father of the person, to whom a girl calls mother, the the brother of that girl calls Mayank his father. Whose photograph is that (relation with Mayank)? Mayank's brother in Law
  • 87. Ram gives to Shyam all the money that he has in his pocket. Shyam adds that much more from his pocket and gives to Ravi, who further adds that much amount and gives that to Kavi, who triples this money and gives to Raju. Raju gives 10 times this amount to Mohan. Mohan gives 90% amount of this to Niru and retains Rs. 120. what is the ratio of this amount to Ram's original amount ? Answer : 10:1 answer
  • 88. The Emperor Augustus, it appears, commissioned an idealized sculpture portrait, the features of which are so unrealistic that they have constituted what one scholar calls an "artificial face." - correct it The Emperor Augustus, it appears, commissioned an idealized sculpture portrait, the features of which are so unrealistic as to constitute what one scholar calls an "artificial face."
  • 89. A recent national study of the public schools shows that there are now one microcomputer for every thirty-two pupils, four times as many than there were four years ago. - correct it A recent national study of the public schools shows that there is now one microcomputer for every thirty-two pupils, four times as many as than there were four years ago.
  • 90. Never before had taxpayers confronted so many changes at once as they had in the Tax Reform Act of 1986. Never before had taxpayers confronted so many changes at once as they in the Tax Reform Act of 1986. (as per past perfect tense, we are using had with the first part – so there is no need with the 2 nd part)
  • 91. Correct it : As have many self-taught artists, Perle Hessing did not begin to paint until she was well into middle age. Like many self-taught artists, Perle Hessing did not begin to paint until she was well into middle age.
  • 92. Statements: I. All pilots are experts. II. All authors are pilots. Conclusions: I. All authors are experts. II. No expert in an author. Only 1 st conclusion is valid conclusion.
  • 93. Statements: I. Some doctors are institutes. II. Some crooks are institutes. Conclusions: I. All institutes are doctors. II. Some institutes are crooks. Only 2 nd conclusion is valid conclusion
  • 94. If the positions of the fifth and the twelfth letters of the word GLORIFICATIONS are interchanged and likewise the positions of the fourth and fourteenth letters, the third and tenth letters, the second and eleventh letters, the first and thirteenth letters, are interchanged, then which of the following will be the twelfth letter from the right end? Answer : T
  • 95. Ostrich is related to Antelope in the same way as Egret is related to (a) Cow (b) Buffalo (c) Camel (d) Zebra Buffalo (both depend on each other)
  • 96. Hong Kong is related to China in the same way as Vatican is related to..?... (a) Canada (b) Mexico (c) North America (d) Rome
  • 97. Solution Hongkong is part of China, similarly Vetican is a part of Rome.
  • 98. Forfeit is related to Surrender in the same way as Remit is related to (a) Perceive (b) Confiscate (c) Exempt (d) Refrain
  • 100. Who is paternal uncle of Pavan? I. Pavan is brother of Poornima, who is daughter of Meena, who is sister of Kumar, who is brother Smrithi. II. Prithvi is brother of Indrajith, who is husband of Poornima, who is mother of Ganga, who is sister of Pavan. From 2 nd statement – it is clear that uncle of Pavan is Prithvi, but 1 st statement doesnt give us this answer.
  • 101. What is Milan’s rank in the class of 44 students? I. Ramesh, whose rank is 17th in the class, is ahead of Shyam by 6 ranks, Shyam being 7 ranks ahead of Milan. II. Suketu is 26 ranks ahead of Milan and Shyamala is 6 ranks behind Milan while Savita stands exactly in the middle of Shyamala and Suketu in ranks, her rank being 17th Answer can be drawn from any statement.
  • 102. Four of the following five have similar relationship and hence form a group. Which one does not belong to the group? (a) BROTHER : DORVEHT (b) ENGLISH : GGNNSIJ (c) ANOTHER : CONVEHT (d) BETWEEN : DTEZEEP (e) HUSBAND:JSUDNAF D is not proper. In Brother – two step to B, OR instead of RO, two step to T, EH instead of HE, two steps to R, we get DORVEHT. Similarly we have to solve all the options.
  • 103. Which of the following relates to FLOWER in the same way as RTERBN relates to SECTOR? (a) RWLGPF (b) EOFKUQ (c) EOFMXS (d) RWLEND R then W, then L, now from behing, E instead of F, N instead of O, D instead of E. We get : RWLEND
  • 104. Who among the following British Governor- Generals shifted India’s capital from Calcutta to Delhi in 1911? a) Lord Louis Mountbatten b) Lord Canning c) Lord Hardinge d) Warren Hastings ANSWER : LORD HARDINGE
  • 105. “ Golden Handshake” is the term associated with a) Share market b) Retirement benefits c) Voluntary retirement benefits d) Smuggling VOLUNTARY RETIREMENT BENEFITSI
  • 106. Which of the following is the first surface- to- surface missile in India? a) Prithvi b) Trishul c) Agni d) Naag PRITHVI
  • 107. Mist is caused by a) Dry ice b) Ice at low temperature c) Water vapours at low temperature d) Carbon- monoxide in solid form WATER VAPOURS AT LOW TEMPERATURE
  • 108. Who among the following was the author of “Rajtarangini”, commonly regarded as the first genuine history of India written by an Indian? a) Banbhatta b) Ravikirti c) Pushpadanta d) Kalhana KALHANA
  • 109. Which of the following Articles of the Indian Constitution deal with the Directive Principles of State Policy? a) 26 to 41 b) 31 to 56 c) 36 to 51 d) 41 to 66 36 TO 51
  • 110. The Chinese pilgrim Fa – Hien visited India during the reign of a) Kanishka b) Chandragupta I c) Chandragupta II d) Harshavardhana Chandragupta II
  • 111. The Indian Navy’s only sailing ship, which returned to Kochi after a 10- month voyage around the globe is, a) INS Vibhuti b) INS Tarangini c) INS Prabhat d) INS Viraat INS Tarangini
  • 112. Correct it : It being an important (a)/ letter, the draft had to be (b)/ seen by the Governor (c)/ itself for approval. It being an important (a)/ letter, the draft had to be (b)/ seen by the Governor (c)/ himself for approval.
  • 113. Correct it : Irrespective of either (a)/ Vijay or Sanjay goes (b)/ the overdue payment (c)/ cannot be collected. (d)/ No error. Irrespective of whether (a)/ Vijay or Sanjay goes (b)/ the overdue payment (c)/ cannot be collected. (d)/ No error.
  • 114. In a certain code language, ‘3a, 2b, 7c’ means ‘Truth is Eternal’; ‘7c, 9a, 8b, 3a’ means ‘Enmity is not Eternal’ and 6a, 4d, 2b, 8b’ means ‘Truth does not perish’. Which of the following means ‘enmity’ in that language? Is and eternal are common in first two lines, so are 7c and 3a. From the first two lines, we find that enmity + not means = 8b+9a from this and third sentence, we find Not is common, which is 8b. Thus 9a is left out, thus ENMITY = 9A (answer)
  • 115. In a certain code language, ‘po ki top ma’ means ‘Usha is playing cards’; ‘Kop ja ki ma’ means ‘Asha is playing tennis’; ki top sop ho’ means ‘they are playing football’; and ‘po sur kop’ means ‘cards and tennis’. Which word in that language means ‘Asha’? In the first 2 sentences IS PLAYING are common, ki ma are also common. Asha + tennis = kop + ja . Let us compare this with po sur kop , where kop is for tennis ( cards and tennis ). Thus Asha = ja (answer)
  • 116. A long rope has to be cut to make 23 small pieces. If it is double folded to start with how many times does it need to be cut? Options : 9,12,11,10 11 as after cutting 11, we are able to get its double 22 pieces – and one last piece – which is at fold (one piece).
  • 117. There are 19 hockey players in a club. On a particular day 14 were wearing the hockey shirts prescribed, while 11 were wearing the prescribed hockey pants. None of them was without either hockey pants or hockey shirts. How many were in complete hockey uniform? 14 + 11 – 19 = 6 answer
  • 118. In a class room three fourth of the boys are above 160 cm in height and they are 18 in number. Also out of the total strength, the boys form only two third and the rest are girls. The total number of girls in the class is 18 are 3/4 th of boys, so total number of boys is : 18 * 4/3 = 24 (total number of boys) of the total strength the ratio of boys and girls is 2:1, so number of girls is 12. answer
  • 119. ‘A’ is east of ‘B’ and west of ‘C’. ‘H’ is South-West of ‘C’, ‘B’ is South-East of ‘X’. which is farthest West? When you put them on a map : B A C (from west to East) south to them is H and X is further North West of B. with the information given, it seems that X is the farthest West.
  • 120. A girl earns twice as much in December as in each of the other months. What part of her entire year’s earning does she earn in December? Let us assume that she earns 1 in each month, but in December she earns 2. Total income = 11*1 + 2 = 13 thus she earns : 2/13 of her total income in December month alone.
  • 121. One watch is 1 minute slow at 1 pm on Tuesday and 2 minutes fast at 1 pm on Thursday. When did it show the correct time? 48 hours have passed and in this duration the watch has gained 3 minutes. We want to know when the watch gained 1 minute. So it should be : 1/3 * 48 = 16 hours. Thus the watch gave correct time on : 1+16 hours = 5 AM on Wednesday answer
  • 122. Which number when placed at the sign of interrogation shall complete the matrix? 6 6 8 5 7 5 4 3 ? 120 126 320
  • 123. solution Here you can see that when you multiply 6*5*4 , you get 120 and so on, thus you have to put 8 in place of interrogation sign (?) so make this matrix perfect. Answer.
  • 124. In the sequence given below the sum of the two digits which immediately precede the digit ‘4′ exceeds the sum of the two digits which immediately follow the digit 4 and sum of the two digits which immediately follow the digit 6 exceeds the sum of the two digits which immediately precede the digit 6. How many such 4’s and 6’s together are there? 54462635642843766483 1) . Let us start with 4 after 54, sum of 54 is 9, and sum of 62 is 8, thus our condition is fulfilled. 2) . 4 (56 =11, 28=10) 3) . 4 (66 * 83) so 3 is answer (only 3 conditions by 4, no condition is fulfilled by any 6)
  • 125. Below are given six three-digit numbers. The digits comprise ofnumeric and letters. The letter indicates its serial order in the English alphabet. What will be the middle digit of the 4th number when the numbers are arranged in the descending order after interchanging numeric in each number without altering the place of letter in the number? 19F, 2H9, 98B, D76, 7A6, 61E We have to interchange numeric first : 91F,9H2,89B,D67,6A7,16E now convert letters into numerics : 916,982,892,567,617,165 Now we have to put them in descending order : 982,916,892,617,567,165 the middle one of 4 th number is A (617 , 1=A) answer
  • 126. In a code language any letter which is immediately after or before a vowel in the English alphabet is substituted by that vowel and any vowel i.e. A, E, I, O and U is substituted by the letter immediately following that vowel in the English alphabet. How can the word FEVERISH be written in that code language? FEVERISH : instead of F, write E, instead of E write F, U for V, F for E, no change in R, J instead of I, no change in S, I for H. So answer : EFUFRJSI answer
  • 127. How many pairs of letters are there in the word SPONTANEOUS which have number of letters between them in the word one less than the number of letters between them in English alphabet? 1). P and T have gap of 2, in alphabet they have gap of : 3. 2). U and S have no gap, in alphabet they have gap of 1. so answer is 2.
  • 128. In the following question one term in the number series is wrong. Find out the wrong term. 11, 5, 20, 12, 40, 26, 74, 54 Here we have two series : 11*2 – 2 = 20 20*2 – 2 = 38 38*2 – 2 = 74 second series is : 5*2 + 2 = 12 12*2+2 = 26 26*2+2 = 54 so instead of 40, we should have 38. answer.
  • 129. More material on reasoning : Click here : http://www.scribd.com/doc/23610014/Reasoning-e-Book
  • 130. Mayank has three children — Sangeeta, Vimal and Ashish. Ashish married Monika, the eldest daughter of Mr. and Mrs. Roy. The Roys married their youngest daughter to the eldest son of Mr. and Mrs. Sharma,, and they had.two children named Amit and Shashi. The Roys have two more children, Roshan.and Vandana, both elder to Veena. Sameer and Ajay are sons of Ashish and Monika. Rashmi is the daughter of Amit. What is the surname of Rashmi ? Sharma (answer)
  • 131. Six lectures A, B, C, D, E and F are to be organised in a span of seven days -from Sunday to Sunday ,-only one,lecture on each day in accordance with the following rules (i) A should not be organised on Thursday. (ii) C should be organised immediately after F. (iii) There should be a gap of two days between E and D. (iv) One day there will be no lecture (Friday is not that day), just before that day D will be organised. (v) B should be organised on Tuesday and should not be followed by D. On which day there is no lecture ? Tentative schedule : S=d M=no T=b W=e T=fF=c S=a On Monday – there is no lecture.
  • 132. There are five persons P, Q, R,S and T. One is football player, one is chess player and one is hockey player. P and S are unmarried and they dont participate in any game. None of the ladies plays chess or football. There is a married couple in which T is the husband. Q is the brother of R and is not a a chess player nor a hockey player. Who is the football player ?
  • 133. Solution P & S dont play so remove them. We have QRT left. One of them is lady, she must be R (as T and Q are men). T and Q play two games - chess and football (as these are not played by ladies). Thus T plays chess and Q plays football and R plays hockey. Answer
  • 134. Of the five boys A, B, C, D and E two are good, one is poor and two are average in studies. Two of them study in post-graduate classes and three in under-graduate classes. One comes from a rich family, two from middle-class families and two from poor families. One of them is interested in music, two in acting and one in sports. Of those studying in under-graduate Classes, two are average.and one is poor in studies. Of the two boys interested in acting, one is a post graduate student. The one interested in music comes from a middle-class family. Both the boys interested in acting are not industrious. The two boys coming from middle-class families are average in studies and one of them is interested in acting. The boy interested in sports comes from a poor family, while the one interested in music is industrious. E is industrious, good in studies, comes from a poor family and is not interested in acting, music or sports. C is poor in studies inspite of being industrious. A comes from a rich family and is not industrious but good in studies. B is industrious and comes from a middle-class family. Name the boy interested in sports.
  • 135. Solution A=rich,non industrius , good in study,acting B=UGmiddle class, industrious , avg. In study ,music C= UGindustrious, poor in study (sports) D= UGmiddle, not-industrious,average in study (acting) E=poor,industrius,good in study, not -(M,S)
  • 136. solution... Thus based on the table, we can say that C is interested in sports.
  • 137. A girl leaves from her, home. She first walks 80 metres in North-west direction and then 30 metres in South-West direction Next, she walks,30 metres in South-east direction. Finally, she turns back towards her house. In which direction is she moving now ? Draw the map and solve it – the direction will be approximately EAST. ANSWER.
  • 138. A walks 10 metres in front and 10 metres to the right. Then every time turning to his left, he walks 5, 15 and 15 metres respectively. How far is he now from his starting point ? DRAW THE MAP AND SOLVE IT ANSWER = 5 METERS.
  • 139. A child is looking for his father. He went 90 metres in the East before turning to his right. He went 20 metres before turning to his right again to look for his father at his uncle's place 30 metres from this point. His father was not there. From here he went 100 metres to the North before meeting his father in a street. How far did the son meet his father from the starting point ? DEDUCT 30 FROM 90 AND 20 FROM 100 FIND SQUARES OF 60 AND 80 AND ADD THEM, YOU GET 10000, SQ.ROOT OF WHICH IS 100. ANSWER IS 100 M.
  • 140. A father tells his son, "I was of your present age when you were born." If the father is 44 now, how old was the boy 5 years back ? THE AGE OF SON IS 22 YEARS NOW. SO HIS AGE 5 YEARS BACK WAS 17 YEARS. ANSWER
  • 141. In a certain code '13' means 'stop smoking' and '59' means 'injurious habit' What is the meaning of '9' and '5' respectively in that code ? I. '157' means 'stop Bad habit'. II., '839'means 'smoking is injurious'. How do you get answer ? Solution : solution can be obtained from any of these two hints. If you take first hint, 59 and 157 have 5 as common number. The common number denotes habit. Thus 5 means Habit. If you take 2 nd hint, 9 is common between 59 and 839 and the common word is injurious, thus 9 means injurious. Thus you can use either of these two statements to answer.
  • 142. WHAT IS THE PUNCHLINE OF BANK OF INDIA RELATIONSHIP BEYOND BANKING
  • 143. WHAT IS THE PUNCHLINE OF CITIBANK CITI NEVER SLEEPS
  • 144. WHAT IS THE PUNCHLINE OF KOTAK MAHINDRA BANK LETS MAKE MONEY SIMPLE
  • 145. WHAT IS THE PUNCHLINE OF SBI WITH YOU ALL THE WAY
  • 146. WHAT IS THE PUNCHLINE OF DENA BANK TRUSTED FAMILY BANK
  • 147. WHAT IS THE PUNCHLINE OF INDIA INFOLINE ITS ALL ABOUT MONEY HONEY
  • 148.
  • 149. WHO IS CEO OF KOTAK MAHINDRA BANK ? UDAI KOTAK
  • 150. WHO IS CEO OF HSBC NAINA LAL KIDWAI
  • 151. WHO IS CEO OF ICICI CHANDA KOCHAR
  • 152. WHO IS CEO OF HDFC BANK ADITYA PURI
  • 153. WHO IS CEO OF AXIS BANK (FORMERLY UTI BANK) SHIKHA SHARMA
  • 154. WHO IS CEO OF STANDARD CHARTERED BANK PETER SANDS
  • 155. WHO IS CEO OF MORGAN STANLEY MUTUAL FUND N. RAMCHANDRAN
  • 156. WHO IS CEO OF INDUSIND BANK RAMESH SOBTI
  • 157. WHO IS CEO OF MUTHOOT GROUP M. GEORGE MUTHOOT
  • 158. WHO IS CEO OF YES BANK RANA KAPOOR
  • 159. WHO IS CEO OF SBI? O.P. BHATT
  • 160. WHO IS CEO OF BARKLEYS BANK MARCUS AGIUS
  • 161. WHO IS CEO OF ABN AMRO BANK GERRIT ZALM
  • 162. WHO IS CEO OF ALLAHABAD BANK OMKAR NATH SINGH
  • 163. WHO IS CEO OF BANK OF AMERICA BRIAN MOYNIHAN
  • 164. WHO IS CEO OF BANK OF RAJASTHAN G. PADMANABHAN
  • 165. WHO IS CEO OF CANARA BANK A.C. MAHAJAN
  • 166. WHO IS CHAIRMAN OF BANK OF MAHARASTRA ALLEN PAREIRA
  • 167. WHO IS CHAIRMAN OF ANDHRA BANK K. RAMCHANDRAN
  • 168.  
  • 169.  
  • 170. A, B, C, D, E, and F are a group of friends. There are two housewives, one professor, one engineer, one accountant and one lawyer in the group. There are only two married couples in the group. The lawyer is married to D, who is a housewife. No woman in the group is either an engineer or an accountant. C, the accountant, is married to F, who is a professor. A is married to a housewife. E is not a housewife. What is E's profession? Males :C (accountant), A, (E) Lawyer females : D(housewife),F (professor), (B) Housewife Thus E is a lawyer, as all other professions are occupied.
  • 171. Five women decided to go shopping to The Loot Bikaner. They arrived at the the Loot showroom in the following order: 1. Archana, 2. Chellamma, 3. Dhenuka, 4. Helen, and 5. Shahnaz. Each woman spent at least Rs.1000. Below are some additional facts about how much they spent during their shopping spree. i. The woman who spent Rs. 2234 arrived before the lady who spent Rs. 1193. ii. One woman spent Rs. 1340 and she was not Dhenuka. iii. One woman spent Rs. 1378 more than Chellamma. iv. One woman spent Rs. 2517 and she was not Archana. v. Helen spent more than Dhenuka. vi. Shahnaz spent the largest amount and Chellamma the smallest. Questions : The woman who spent Rs. 1193 & when did she come? A=1340, C = 1193, S=2573, H= 2517,D=2234 1193 is smallest, so it was spent by Chellamma. She is 2 nd to come. Answer
  • 172. Let A and B be two solid spheres such that the surface area of B is 300% higher than the surface area of A. The volume of A is found to be k% lower than the volume of B. The value of k must be
  • 173. solution Surface area of sphere : 4pi*r*r if we assume smaller sphere of R=1 and larger sphere of R=2, then our condition is fulfilled. Now find volume of the two spheres : 4/3*pi*r*r*r so the ratio of the bigger and smaller spheres are : 8:1 thus volume of A is lower by : 7/8*100 = 85% anwer
  • 174. There are 8436 steel balls, each with a radius of 1 centimeter, stacked in a pile, with 1 ball on top, 3 balls in the second layer, 6 in the third layer, 10 in the fourth, and so on. The number of horizontal layers in the pile is Solution : first pile : 1, 2 nd pilw ; 6 3rd pile : let us add the series : 1+3+6+10+15+21+28+36...... there are two series : 1, 1+2, 3+3,6+4 .... ,
  • 175. The 288th term of the series a, b, b, c, c, c, d, d, d, d, e, e, e, e, e, f, f, f, f, f, f.... should be : We can see the series is : 1,3,4, (a is 1, b is 2, c is 3 ,,, so on apply formula of Total of arithematic progression: Tn = n/2(2A+(n-1)d) 288=n/2 (2+nd-d) use options to solve questions earliers.
  • 176. A shopkeeper increases price of an item by 20% and then gives a discount of 20% on new price. What is the net impact ? Let us assume that initial price is 100, new price is 120 now 20% discout is applied from 120 120 -24 = 96 thus we can see that there is a net impact of - 4%. in short : 20% multiplied to 20% gives -4% ans.
  • 177. There are certain words written randomly, can you infer what will come next : A8R L24M X72H ....? Try to find some pattern : A = 1, L = 12, X = 24 next : K numberic : 8 * 3 = 24, now 24*3 = 72, next :216 R = 18, M = 13, H 8, next : C we can say next code : K216C answer
  • 178. We use + for multiply, - for divide, / for subtraction and * for addition. Following BODMAS, what should be the answer : for the following : 3* 5-1/4+2 = answer : 0 2* 3* 4/ 2 – 1 +2+2 = =1 answer 2-2+3-4/ 4 *2 = =- 5.25 answer
  • 179. 20 persons came to a party and each person shook hand with each other person. How many handshakes were there? Start with small number to validate and understand. Suppose there are 4 persons, first will shake hand with 3, second will shake hand with 2, third will shake hand with 1. we have : 3+2+1 = 6. thus we can say : N * (N -1) / 2= 6 similarly here : (20 * (20-1)/ 2 = 190 answer
  • 180. How many zeroes are there when you count from 1 to 100? One each from 10 to 90 : + 2 in 100 = 11 zeros.
  • 181. You take up all those positive numbers which are ending with 2 and are less than 100 and are divisible by 3. How many such numbers are there? 12, 42, 72 thus we have 3 such numbers.
  • 182. We have 0,1,2,3,4 as digits. How many 5 digit numbers can we make out of these. Repeatition is not allowed? For the first number, you have 4 options (zero not be used) : for 2 nd number you have 4 options again (the first number will not be used) , for 3 rd number we have 3 options, so on = 4*4*3*2*1 = 96 answer
  • 183. Raju had to add two digits, but by mistake he multiplied these two digits and got 56. He was again asked to add these two digits, but this time he squared up these numbers and found their difference, it was 15. what will you get if you find the correct answer ? X* Y = 56 X^2 – Y^2 = 15 (x+Y) (X-Y) = 15 now try with options at this stage. Take factors of 56, which are : 2*2*2*7 the factors could be : 4*14 or 8*7 so X = 8 and Y = 7, their difference is 1 and X +Y = 15, so the correct answer is 15. answer .
  • 184. In order to go to his office from home, Vivek goes first to his right then he turns towards left three times. Then he again turns to his right. He has to again turn left 2 times to enter his office, which is North facing. What direction is his house facing? Total left turns : 3 + 2 = 5 total right turns : 1 + 1 = 2 net : 3 left turns. Now from office, he will take net 3 right turns to enter his home. He will be west facing while entering home, so home will be East facing. Answer : east facing.
  • 185. Kapil says that his birthday is after 26 February, but his friend Mayank says that it is before 1 March. His another friend Vivek says that it is between 25 and 28. When is the birth day of Kapil? 27 February. Answer
  • 186. Mayank swims across a river a distance of 200 meters 12 minutes, but he can also swim with the current in water same distance in 5 minutes. What is his speed in still water? Formula = (Distance ) / (Time ) = speed while going accross the river, the speed is : X-Y= a ( 200/12)= 16.66 , where X is Mayank's speed and Y = water's speed while with the water, the speed is X + Y = b (200/5) = 40 the speed of Mayank is average of 40 and 16.66 = 28.33 and the speed of water is (40-16.66)/2 = 11.66 m. Per minute. answer
  • 187. WHO HAS TAKEN HIGHEST NUMBER OF WICKETS IN 1 DAY AND TEST CRICKET? M . MURALIDHARAN OF SRILANKA
  • 188. WHERE WILL NEXT OLYMPICS BE HELD? LONDON 2012
  • 189. NAME OF CAPTAIN OF HOCKEY TEAM IN INDIA ? MALE : SANDEEP SINGH COACH : MAHARAJ KISHAN KAUSHIK FEMALE : SURAJ LATA
  • 190. WHICH COUNTRY PRODUCES MAXIMUM GOLD ? SOUTH AFRICA (AND INDIA CONSUMES THE MOST)
  • 191. WHO IS THE PRIME MINISTER OF UK? TONY BLAIR
  • 192. HOW MANY COUNTRIES ARE MEMBERS OF WTO NOW ? 153 WTO = World Trade Organisation , earlier called GATT, set up in 1995.
  • 193. What is the nickname of Glen McGrath? Pigeon
  • 194. A,B,C speak one statement true and one false. A says to B : “I am a thief.” 2. “You are a Doctor.” B: “Yes, I am a doctor” “C is a thief.” C adds : 1. “ I am a theif.” “B is a Doctor.” Who is theif ? Let us suppose A's 1 st statemet is right, 2 nd statement is wrong. Thus B's 1 st statement is false and 2 nd right. C's 2 nd statement is false, thus 1 st is correct. Here there are two theifs – A and C. If we assume A's 2 nd statement to be correct, then A is not a theif, and no one is a theif. Thus either no one is theif or A+C both are theives. Logically , NO ONE is theif here.
  • 195. A had to buy some nuts. He buys half the nuts required. He takes them and his son steals half of these. Out of remaining, his daughter steals half. Now when A is fired by his boss that he had brought exactly 70 less than what was required. What is the ratio of nuts brought and to be brought? Suppose he buys 24 nuts instead of 48. his son takes away 12 and his daughter takes away 6 out of the remaining. He finally has 42 less than required. Thus original requirements : 70/42* 48 = 80 thus ratio is : 1:8 answer (you may assume any numbers but answer will be same).
  • 196. A,B,C each deposit Rs. 1000 in a partnership. Out of this A takes away half of his capital after 6 months, B takes away 25% of capital 4 months before the year end. And C doubled his money after 3 months. Out of profit of 82000 how much will A get? A _ (6*1000) + (6*500) = 9000 B= (8*1000) + (750*4) = 11000 c = (3*1000) + (9*2000) = 21000 their profit sharing ratio : 9:11:21 A will get : 9/(9+11+21) * 82000 = 18000 answer.
  • 197. Who is Noble peace prize winner for 2009? Barak Obama
  • 198. Who won booker award in 2009? Hilary Mantel
  • 199. Who is the president of United Nations ? Dr. Ali Abdussalam Treki
  • 200. How many seats did congress win in the last national election? 206 out of UPA 259 seats out of total 543
  • 201. Who won the filmfare award for best film in 2008? 53 rd filmfare award for best movie : tare zemeen pe
  • 202. Who won the US open (female) in Tennis? Justine Henin
  • 203. Name the books written by Chetan Bhagat ? 5 point someone one night at call centre 3 mistakes of my life 2 states – a story of my marriage
  • 204. Name the MD of IMF? Dominique Strauss-Kahn
  • 205. WHO IS GOVERNOR OF RBI ? Dr. D. Subbarao
  • 206. WHO IS THE DIRECTOR OF 3 IDIOTS? RAJKUMAR HIRANI
  • 207. WHO WON THE DATASAHEB PHALKE AWARD 2008? cinematographer V K Murthy
  • 208. NAME AUTOBIOGRAPHIES OF INDIAN CRICKETER GAVASKAR ? Sunny Days : Sunil Gavaskar
  • 209. HOW MANY CENTURIES HAVE BEEN HIT BY TENDULKAR ? 45 IN ONE DAY 46 TEST MATCHES (46TH AGAINST BANGLADESH ON 21 DECEMBER 2009)
  • 210. Which of these is correct ? a. Having completed his lunch, a dog bit Mohan, who went to market. b. Having completed his lunch, Mohan went to market, where he was bite by a dog. c. Having completed his lunch, Mohan went to market, where he was bitten by a dog. Answer : C.
  • 211. Which of these is correct? 1. although he worked very hard, but he has failed. 2. although he had worked hard, yet he had been failed. 3. although he worked very hard, yet he failed. Answer : 3
  • 212. Which of these is correct? 1. He reached station before the train had reached 2. he had reached the station, where train had reached after him. 3. he had reached the station before the train. Answer : 3.
  • 213. Fill in the blank : I prefer tea _____ coffee options : from, then, than, to, on ANSWER : TO
  • 214.
  • 215.
  • 216. Ram and Shyam had equal money, which they deposited in two different banks. One got 10% simple interest, but the other got 5% half yearly compounding interest. At the end of 2 years, their amounts had difference of Rs. 10, what was the original amount deposited by them? Let us suppose they deposited 100 each. First person will get : 100+10+10 = 120 2 nd person will get : 100 + 5 + 5.25 + 5.5125 + 5.79 or 5.80 = 121.56 the difference is 1.56, if difference is 10, the principal should be : 10/1.56* 100 = 600 by each approx.
  • 217. A shopkeeper uses weight of 900 grams instead of 1000 grams (1kg) while selling. He sells at his cost + 10%, what is his actual profit in %? He sells 900 grams when he is charging for 1000 grams. Suppose he purchased 900 for Rs. 900. His cost is 900, but he is charging for 1000 + 10% = 1100 his profit is 200 on 900, or 200/900 *1000 = 22.2% answer
  • 218. A circle (of maximum size possible) is inserted inside a squre. How much of the area of square is still left out of total area in %? Let us suppose that the side of square is 14. area of square is 14*14 = 196 area of circle : 22/7*7*7 = 154 area left : 196-154 = 42 in % = 42/196 *100 = 21% approx. Answer you can assume any value instead of 14 and try to work out the %, the answer will be the same.
  • 219. Out of the following chart, please, tell how many female students got between 50 - 60% in PGPSE ?
  • 220. Solution... We can see that 12 girls got more than 60% marks. 40 – 12 – 2 = 26 got between 50 to 60%. answer
  • 221. Based on the data given below, find out sale of Raju TV store. 1990 was the first year, when Raju TV sold 100 TV. Every person who buys TV buys again after 4 years. Raju sells 400, 600, 800 TV in 94, 98, 99 respectively. There was no new customer in 2000, but the sale was simple average of sale of 1990 and 1994. How many TV did Raju TV sold in 1996.
  • 222. solution.... Average of 1990 and 1994 = 250 no new customer in 2000, so 250 TV will be sold as repeat customers. Thus sale in 1996 = 250 TV.
  • 223. Ram, Ajay and Ravi (3 friends) are experts in three games – cricket, hockey and golf. One of these live in Delhi, and other two live in Mumbai. The person who lives in Delhi doesnt like Golf. Hockey player lives in Mumbai. The person whose name is smallest is also the best player of Golf. Ajay daily goes to his friend's home. Ravi has to visit Mumbai twice a year. Who plays cricket? Ajay lives in Mumbai (he cant be Delhi, as he daily visits his friend). The person who lives in Delhi doesnt like Golf, he also cant play Hockey (Hockey player is in Mumbai), so that persn plays Cricket. So the person who lives in Delhi plays criciket. Ravi visits Mumbai,so he lives in Delhi. Ravi plays Cricket.
  • 224. Nanu's maternal grand father, and maternal grand mother both are doctors. His father is Engineer, but his mother is Professor. A is elder to C, but Younger to B. D is wife of A. E is husband of B. The eldest person is a male but he is neither A nor Nanu. Out of two ladies, the elder is Doctor and younger is professor. Who is Engineer. Which statement is redudant? A is engineer (as he is the second male in hierarchy). The last statement : out of two ladies, the elder is doctor and younger is professor, is redudant – as it is not required).
  • 225. 118, 66, 252, 137 .... Look at the series, you find that 66 has been computed from 118 as under : 1^3+1^3+8^2 = 66, similarly 6^3 + 6^2 = 252 so : 137 = 1^3 + 3^3 +7^2 = 77 answer
  • 226. What will come next :;;; 1,8,27,64... so on??? 1^3 = 1, 2^3 = 8, 3^3 = 27, 4^3 = 64, 5^3 = 125, thus answer is 125.
  • 227. A went to buy 2 shirts and 1 trousers. He carried Rs. 1000 with him (just sufficient) . The ratio of prices of shirt and trouser is : 7:6. however, he purchased 2 trousers and 1 shirt and returned back with Rs. 50 also. What will he pay, if he buys 2 shirts and 3 trousers? 2 Shirt + 1 Trousers = 1000 1 shirt + 2 trousers = 950 1 shirt – 1 trousers = 50, Now from 1 st equation: 2 trousers + 1 trousers + Rs. 100 = 900 thus 1 trousers = 300 and 1 shirt = 350. he will spend 700+900 = 1600 answer.
  • 228. X has to organise his lecture plan. He has to teach entrepreneurship, Planning, Finance, and Law on 4 days and one day is for IT. IT must be before Law, but Planning should not be on the 1 st day. Finance and IT must be on any of the last 3 days. Prepare complete schedule. Solution: Last day : law, 2 nd last IT, 3 rd last : Finance, 4 th last : Plannning; the first day : Entrepreneurship. It is better to solve the last day first in this types of questions. Answer.
  • 229. A,B,C,D are working on a project with different names. W is not A, B is not Y or Z. D and E have no relation to C and W Who is A? Answer cannot be determined / data inadequate
  • 230. There are two numbers whose HCF and LCM are 8 and 25, what are those numbers? No such numbers are possible as LCM 25 is not divisible by 8, LCM is always divisible by HCF. Answer
  • 231. Mayank Bothra wants to bring tiles for his room. His room's size is 28 feet by 20 feet. What should the size of square tile, so that minimum number of tiles are used? Find HCF of 28 and 20. The HCF is 4, so the size of tile should be 4 feet by 4 feet. The total number of tiles required will be : (28*20)/(4*4) = 35 tiles answer
  • 232. A monkey is trying to climb a Date tree. He climbs 4 feet in 2 minutes, but in the next 1 minute, he slips by 3 feet. When will he reach at the top of this Date tree which is 36 feet high? Actually he travels 1 feet in 3 minutes. He has to travel (36-4) = 32 feet, which will take 32*3 = 96 minutes. Then in the next 2 minutes he will be at the top of the tree. Thus he will take 98 minutes in all.
  • 233. Mayank Bothra buys apples @ 4 for Rs. 3 and sells them @ 3 for Rs. 4. what is his profitability? There are two important numbers 3 and 4, so let us take LCM of 3 and 4, this is 12. let us assume Mayank got 12 apples. He had to pay 9 to buy 12 apples. Now he goes to sell them, he gets Rs. 16 when he sells them. His profit is (16-9) – 7, so his profitability is 7/9 *100 = 77.77% answer
  • 234. Naresh Jain makes payment to a shopkeeper. By mistake the shopkeeper reveses the digits of the amount to be paid and takes Rs. 54 more than what was required. What was the actual payment due? Hit and try for this type of question. Try for 39 and 93. 93-39 = 54. The shopkeeper has taken 93 instead of 39. So the amount due was 39. answer
  • 235. A and B run opposite to an elevator. The ratio of the speed of elevator, A and B is : 1:6:3 Elevator is going up, but A and B are coming down. A takes 1 minute and B takes 2.5 minutes to come down. How much time will they take if elevator was not moving? If elevator is not moving, they will take less time. Look at the speed ratios. A will take 1/6 time less – so he will take 60 seconds – 10 seconds= 50 seconds and B will take 1/3 time less, so he will take : 150 seconds – (150*1/3) = = 100 seconds. Answer. 2.5 minutes = 150 seconds
  • 236. Ajay has to pay to Mayank Rs. 1100 after 1 year. What should he pay now and settle the accounts? Rate of interest is 10% We have to find the present value of 1100 formula of present value is (Amount * 100)/ (100 + rate of interest * no. Of years) = (1100 * 100) / (100 +100) = 1000
  • 237. A, B,C, & D start a game. In the beginning all of them had equal amount. The winner has to collect half the amount of all other players. In the first round C wins. In the 2 nd round B wins and in the 3 rd round B again wins. At this time, their scores are : A : 50, B : 1250, C : 250 D : ? What was the score of each of these in the beginning ? The score of D should be equal to that of A = 50. Now add the score of all of these, we get 1600. Thus each of these should have score of 1600/4 = 400 answer
  • 238. A thief steels a necklace and runs @ 10 Km per hour at 3 pm. After 18 minutes, a policeman starts chasing him @ 15 km per hour. Whenwill the thief be caught? At the speed of 10, the thief would have run 10 *18/60 = 3 Km. So policeman has to chase him for this distance to start : 3 / (15-10) * 1/60 =
  • 239. Mayank Bothra wants to get his room coloured. His room is 10 Ft by 20 Ft. Assuming that colouring costs Rs. 100 per sq. ft., how much will Mayank pay (for four walls), height of walls is 10 feet? The side towards length : 20*10 = 200 the side towards width : 10*10 = 100 total 4 walls (200*2+100*2 )=600 ft. Total payment : 600*100=Rs. 60000 ans.
  • 240. Rishi has put his icecream in a rectangular bowl of 10*20 inch with depth of 5 inch. Now he wants to make a round ball out of this. What will be the diameter of this ball? Total quantity of icecream = 10*20*5 = 1000 cubic inch volume of sphere : = 4/3*pi*radius^3 = 4/3 22/7*r*r*r = 1000 r= cubic root of (21000/88) r= 6.5 approx. Or diameter= 13 inch approx. Answer
  • 241. In a typcial coding exercise, a person says that RAM= 32, and SITA= 49, what will be KRISHNA In that code? RAM = R= 18, A=1, M = 13 so adding these RAM = 32 SITA, S=19, I=9, T=20,A=1 Adding these we get 49, so KRISHNA should be : K=11,R=18,I=9,S=19,H=8,N=14,A=1 TOTAL = 80 ANSWER
  • 242. If Earth:Moon, and Sun:mercury, what should be X: Titan The Moon rotates round the Earth, Mercury rotates round the Sun, Titan rotates round the Saturn. So X = Saturn.
  • 243. Rishi has a white cube of 5 inch dimensions. He cuts it to make smaller cubes of 1 inch each. Before this he dips the cube to colour one of its dimension as red. Now how many cubes will have red as a colour on one side? The cube is cut into dimension of 1 inch each. Thus (5*5*5)/(1*1*1) = 125 thus we have now total 125 small cubes. Out of these 125, 25 cubes are such which have one dimension as red.
  • 244. A group leader puts his group in rows and columns. He finds that the number of rows and columns are equal. He finds that 20 more persons have to come. If he puts these 20 persons, there is one more complete row or one more column. What is the number of persons in his group? It is clear that there are 20 rows and 20 columns, so total number of persons is 400. add 20 more persons to it, you get : 20*20 = 400 + 20 = 420 answer
  • 245. Mayank has some money in his pocket. He gives me half of this. Now he spends 75% of the remaining on purchasing a shoe. He then realises that he has to buy a T shirt also. So he spends 80% of the remaining on a T-shirt. He is now thirsty and so spends 20% on a mineral water bottle. He is left with Rs. 40. How much did he have in the beginning? Start from last, multiply each value by 100/ (100-what % is spent) : 40*100/80 = 50 50*100/20 = 250 250*100/25 = 1000 1000*100/50 = 2000 thus he had 2000 in the beginning
  • 246. Mayank Bothra has prepared a software, which puts the letters in alphabetic order, but one at a time in 1 second. How much time will the software take to put T I G E R in ascending or descending order? T I G E R first of all E will be put in the beginning then G will be put after E. Then I will be put after G and then R will be put before T. Total time : 4 seconds. In descending order : 1 second only (as only R has to be put after T)
  • 247. What is BODMAS? B= BRACKET O = OF D=DIVISION M=MULTIPLY A = ADDITION S=SUBTRACT apply these in this sequence only. So 9+9/9-2*1of (2-3*1/3) = 9+1-2= 8
  • 248. Which is bigger : 12/13 14/16 16/19 23/29 let us compare 12/13 and 14/16, logically 12/13 should be bigger (gap between 13 and 12 is only 1). still : cross multiply : 12*16 and 13*14 , which is bigger? = 12*16 is bigger so 12/13 is bigger. Similarly we have to compare all the values and find the biggest number.
  • 249. What will come next in the series : ANW, DKT, GFQ, ....? The series is : A – D – G so next digit should be : J (gap of + 2) next series is : N-K-F so next : Y (increasing gap -2,-4,-6) next series : W-T-Q = N (gap of -2) so answer : JYN answer
  • 250. A goes from X to Y at 10 am and will reach at 12 and B goes from Y to X at 9 and will reach at 12. when will they cross each other? A travels for 2hours, B for 3 hours. Let us assume that the distance is (2*3) = 6 KM. A travels @ 3 KM per hour and B @ 2 km per hour. At 9 B starts, till 10 he covers 2 KM. Now gap between A and B is 4 Km. Divide 4 by (2+3) = 4/5 = .8 , so they will meet at 10 + (.8 * 60) = 48 minutes. So They wil meet at 10 + 48 min. Ans
  • 251. A puts an item at 10% discount, before doing this he had actually increased the marked price by 10%, what is the change in his revenue? Let us assume that the earlier marked price is 100. add 10% = 110, now A reduces it by 10% (10% discount on marked price) so he gets 110-11 = 99 thus there is 1% reduction in his revenue or (10% * 10% = 1%)
  • 252. A takes up 2 types of rice – one at 10 per kg and another at 50 per kg, how should he mix these to get a profit of 20% when he sells them @ 30 per KG? Sale price is 30, so cost should be 30*100/120 = 25 in order to get the ratio of mixture, let us deduct 25 from 10 and 50, we get : 15 and 25, now reverse these ratio : 25:15 or 5:3, thus rice of the rate 10 should be 5/(5+3) = 5/8 and rice of rate 50 should be 3/8 or their ratio should be 5:3 answer.
  • 253. A and B are partners and share profit in the capital ratio of 3:2, however, A withdraws his half of the money after 5 months, and B doubles his money after 8 months, what is the final profit sharing ratio? For first six months, the capitals are in the ratio of 3:2, but now there is some change. A withdraws 1.5 and after 2 more months, B adds 2 more. Thus (3*6months + 1.5*6months) : (2*8 months + 4*4months) 27: 32 answer
  • 254. A:B = 2:3, C:D = 3:4, B:D = 3:2, what is A:C? In A:B and B:D, we have B common, so A:B:D = 2:3:2 We know that C:D is 3:4, so keep the value of D as 4, thus new ratio is : A:B:C:D = 4:6:3:4 thus A:C = 4:3 answer
  • 255. A can complete a work in 20 days and B can do the same work in 30 days. Both work together for 1 day and get 500 as total amount, how should this money be divided between A and B? A: B should be 30:20 (reverse of 20:30 – the number of days taken by them in doing the work). Thus A should get : 300 and B 200 answer.
  • 256. A,B,C start a work together. A can do it alone in 20, B in 30 and C in 40 days, in how many days will they finish the work? A's one day work : 1/20, B's one day's work : 1/30 and C's one day's work : 1/40, add them : 1/20 + 1/30 + 1/40 (6+4+3)/120 = 13/120 , now reverse it : 120/13 = 9.2 days or 10 days approx.
  • 257. There are 720 students in an examination, out of them, 1/3 rd fail in English, 1/4 th fail in Mathematics, 1/8 th fail in both the paper. How many dont fail in any paper? Out of 720, 240 fail in English, and 180 fail in Mathematics and 90 fail in both the paper. Thus the total number of students who have failed in either paper : 240+180 – 90 = 330 thus the students who have not failed in any paper is : 720 – 330 = 390 answer
  • 258. Kapil is 4 th from top and 106 th from bottom. How many students are there in total ? There are three students before Kapil, 105 below him and he himself so : 3+105+1 = 109 or 4+106 – 1 = 109 answer
  • 259. A and B have salary in the ratio of 10:11, but their savings are in the ratio : 3:7, but now their salaries increase by 100% each. If earlier their total savings was Rs. 2100, what is their new savings, if they dont change their expenditure and they spend equal amounts? Their total savings : 2100, so A saves 3/10 *2100 = 630 and B saves : 7/10 * 2100 = 1470 difference of their savings is : 840, so A should earn 840 *10 = 8400 and spend (8400-630) = 7770 and B should earn 9240 and save (9240-1470) = 7770. thus their new incomes are : 16800 and 18480 and new savings are : (16800 – 7770) = 9030 and (18480-7770) = 10710 answer
  • 260. A and B run a race and A gives B a start of 100 meters in 1000 meter race, still A wins by 1 second or 1 meters. What is the ratio of their speeds? B takes 1 second to cover 1 meters, so his speed is 3600 meters per KM or 3.6 km per hour. He has actually run 900 meters only. He should take 900/3600*60 = 15 minutes A has run 1000 meters in 14 minutes and 59 seconds. So his speed is 40 KM per hour approx. Thus the ratio of their speed is 40:36 or 10:9 answer
  • 261. What is 10% of 30% of 1500? 30% of 1500 = 450 now 10% of 450 = 45 so answer is 45
  • 262. Which is more 3^1/3 or 4^1/4? Let us take the two powers : 1/3 and 1/4, let us now get same powers. LCM of 3,4 is 12, so let us multiply them by 4/4 and 3/3, we get : 4/12 and 3/12. we can now solve the numerator powers : so answer is : 81^1/12 and 64^1/12 now we can see that 81^1/12 is bigger, so the answer is 3^1/3 is bigger than 4^1/4
  • 263. Ravi sees a photo and says that “this is the photo of younger son of the brother of the grandson of the father of the brother of the mother of my sister”, whose photo was it? Answer : This is the photo of nephew of Ravi (son of Ravi's cousin) answer In order to solve such questions, start from last, and try to solve each relationship in simple terms or prepare a tree diagram from these statements. For example, : brother of the mother of my sister = maternal uncle – so similarly replace all these sentences.
  • 264. MORE LINKS TO DOWNLOAD PRACTICE MATERIAL: http://www.scribd.com/doc/23407950/mathematics-for-OPENMAT-MAT-CAT-GMAT
  • 265. ADDITIONAL LINKS http://www.scribd.com/doc/11692763/Advanced-Mathematics-for-GMAT-CAT-MAT http://www.scribd.com/doc/23407929/Advanced-Mathematics-for-GMAT-CAT-MAT http://www.scribd.com/doc/11625819/Basic-Mathematics-for-Cat-GMAT-Mat http://www.scribd.com/doc/23407954/mathematics-for-OPENMAT-MAT-CAT-GMAT-25-april http://www.scribd.com/doc/23407934/Advanced-Mathematics-for-GMAT-CAT-MAT2 http://www.scribd.com/doc/6583520/English-Afterschoool-21-May-2 http://www.scribd.com/doc/23300964/Advanced-Mathematics-for-GMAT-CAT-MAT2 http://www.scribd.com/doc/23407945/MATHEMATICS-FOR-ATMA-CAT-MAT-GMAT-BANK-PO-GRE
  • 267. http://www.scribd.com/doc/23393680/REASONING-AFTERSCHOOOL http://www.scribd.com/doc/23393719/reasonning-44 http://www.scribd.com/doc/6583347/DI-and-Reasoning http://www.scribd.com/doc/23407636/10-July-reasoning http://www.scribd.com/doc/6583273/Reasoning-Quiz http://www.scribd.com/doc/23393476/10-July-reasoning http://www.scribd.com/doc/23393716/REASONING-QUIZ http://www.scribd.com/doc/14705025/17-Reasoning-II http://www.scribd.com/doc/23393478/10-July-reasoning-II
  • 269. THANKS.... GIVE YOUR SUGGESTIONS AND JOIN AFTERSCHOOOL NETWORK / START AFTERSCHOOOL SOCIAL ENTREPRENEURSHIP NETWORK IN YOUR CITY [email_address] PGPSE – WORLD'S MOST COMPREHENSIVE PROGRAMME IN SOCIAL ENTREPRENEURSHIP